Индукция магнитного поля формулы: Магнитная индукция, магнитный поток: определение, формулы, смысл

Содержание

Магнитная индукция, магнитный поток: определение, формулы, смысл

Магнитная индукция (обозначается символом В) – главная характеристика магнитного поля (векторная величина ), которая определяет силу воздействия на перемещающийся электрический заряд (ток) в магнитном поле, направленной в перпендикулярном направлении скорости движения.

Магнитная индукция определяется способностью влиять на объект с помощью магнитного поля. Эта способность проявляется при перемещении постоянного магнита в катушке, в результате чего в катушке индуцируется (возникает) ток, при этом магнитный поток в катушке также увеличивается.

Физический смысл магнитной индукции

Физически это явление объясняется следующим образом. Металл имеет кристаллическую структуру (катушка состоит из металла). В кристаллической решетке металла расположены электрические заряды — электроны. Если на металл не оказывать ни какое магнитное воздействие, то заряды (электроны) находятся в покое и никуда не движутся.

Васильев Дмитрий Петрович

Профессор электротехники СПбГПУ

Если же металл попадает под действие переменного магнитного поля (из-за перемещения постоянного магнита внутри катушки — именно перемещения), то заряды начинают двигаться под действием этого магнитного поля.

В результате чего в металле возникает электрический ток. Сила этого тока зависит от физических свойств магнита и катушки и скорости перемещения одного относительно другого.

При помещении металлической катушки в магнитное поле заряженные частицы металлический решетки (в кашутке) поворачиваются на определенный угол и размещаются вдоль силовых линий магнитного поля.

Чем выше сила магнитного поля, тем больше количество частиц поворачиваются и тем более однородным будет являться их расположение.

Магнитные поля, ориентированные в одном направлении не нейтрализуют друг друга, а складываются, формируя единое поле.

Формула магнитной индукции

где, В — вектор магнитной индукции, F — максимальная сила действующая на проводник с током, I — сила тока в проводнике, l — длина проводника.

Магнитный поток

Магнитный поток это скалярная величина, которая характеризует действие магнитной индукции на некий металлический контур.

Магнитная индукция определяется числом силовых линий, проходящих через 1 см2 сечения металла.

Магнитометры, используемые для ее измерения, называют теслометрами.

Абрамян Евгений Павлович

Доцент кафедры электротехники СПбГПУ

Единицей измерения магнитной индукции в системе СИ является Тесла (Тл).

После прекращения движение электронов в катушке сердечник, если он выполнен из мягкого железа, теряет магнитные качества. Если он изготовлен из стали, то он имеет способность некоторое время сохранять свои магнитные свойства.

Формула индукции магнитного поля, B

Направлением вектора магнитной индукции считают направление на север магнитной стрелки, которая может свободно вращаться в магнитном поле. Такое же направление имеет положительная нормаль к замкнутому контуру, по которому течет ток. Положительная нормаль имеет направление, совпадающее с направлением перемещения правого винта (буравчика), если его вращают по направлению тока в контуре.

Модуль вектора магнитной индукции можно установить, используя силу, которая действует на проводники с током, помещенные в магнитное поле (силу Ампера). Тогда модуль вектора равен частному от деления максимальной силы (), с которой магнитное поле оказывает воздействие на отрезок проводника с током (I) к произведению силы тока на длину проводника ():

   

Рассматривая силу Лоренца, которая действует на заряженную частицу, движущуюся в магнитном поле, получают формулу для магнитной индукции в виде:

   

где – модуль силы Лоренца; q – заряд частицы, движущейся со скоростью v в магнитном поле; – это угол между векторами и .

Направления , векторов и связаны между собой правилом левой руки.

Формулой, которая определяет величину вектора магнитной индукции в данной точке магнитного поля, считают так же следующее выражение:

   

где – максимальный вращающий момент, действующий на рамку, которая обладает магнитным моментом , равным единице, если нормаль к рамке перпендикулярна направлению поля. Вращающий момент (M), действующий на контур с током I в однородном магнитном поле можно вычислить как:

   

где S – площадь, которую обтекает ток I. Следует помнить, что максимальный вращающий момент получается тогда, когда плоскость контура параллельна линиям магнитной индукции поля ().

Принцип суперпозиции магнитных полей

Если магнитное поле получается в результат наложения нескольких магнитных полей то, магнитная индукция поля (), может быть найдена как векторная сумма магнитных индукций отдельных полей ():

   

Закон Био-Савара-Лапласа, как формула для вычисления величины индукции магнитного поля

Закон Био-Савара – Лапласа является одним из распространенных законов, который позволяет вычислить вектор магнитной индукции () в любой точке магнитного поля, создаваемого в вакууме элементарным проводником с током:

   

где I – сила тока; – вектор элементарный проводник по модулю он равен длине проводника, при этом его направление совпадает с направлением течения тока; – радиус-вектор, который проводят от элементарного проводника к точке, в которой находят поле; – магнитная постоянная.

Вектор является перпендикулярным к плоскости, в которой расположены и , конкретное направление вектора магнитной индукции определяют при помощи правила буравчика (правого винта).

Для однородного и изотропного магнетика, заполняющего пространство, вектор магнитной индукции в вакууме( и в веществе (), при одинаковых условиях, связывает формула:

   

где – относительная магнитная проницаемость вещества.

Частные случаи формул для вычисления модуля вектора магнитной индукции

Формула для вычисления модуля вектора индукции в центре кругового витка с током (I):

   

где R – радиус витка.

Модуль вектора магнитной индукции поля, которое создает бесконечно длинный прямой проводник с током:

   

где r – расстояние от оси проводника до точки, в которой рассматривается поле.

В средней части соленоида магнитная индукция поля вычисляется при помощи формулы:

   

где n – количество витков соленоида на единицу длины; I – сила тока в витке.

Примеры решения задач по теме «Индукция магнитного поля»

Магнетизм — Физика — Теория, тесты, формулы и задачи

Оглавление:

 

Основные теоретические сведения

Сила Ампера

К оглавлению…

Заряженные тела способны создавать кроме электрического еще один вид поля. Если заряды движутся, то в пространстве вокруг них создается особый вид материи, называемый магнитным полем. Следовательно, электрический ток, представляющий собой упорядоченное движение зарядов, тоже создает магнитное поле. Как и электрическое поле, магнитное поле не ограничено в пространстве, распространяется очень быстро, но все же с конечной скоростью. Его можно обнаружить только по действию на движущиеся заряженные тела (и, как следствие, токи).

Для описания магнитного поля необходимо ввести силовую характеристику поля, аналогичную вектору напряженности

E электрического поля. Такой характеристикой является вектор B магнитной индукции. В системе единиц СИ за единицу магнитной индукции принят 1 Тесла (Тл). Если в магнитное поле с индукцией B поместить проводник длиной l с током I, то на него будет действовать сила, называемая силой Ампера, которая вычисляется по формуле:

где: В – индукция магнитного поля, I – сила тока в проводнике, l – его длина. Сила Ампера направлена перпендикулярно вектору магнитной индукции и направлению тока, текущего по проводнику. 

Для определения направления силы Ампера обычно используют

правило «Левой руки»: если расположить левую руку так, чтобы линии индукции входили в ладонь, а вытянутые пальцы были направлены вдоль тока, то отведенный большой палец укажет направление силы Ампера, действующей на проводник (см. рисунок).

Если угол α между направлениями вектора магнитной индукции и тока в проводнике отличен от 90°, то для определения направления силы Ампера надо взять составляющую магнитного поля, которая перпендикулярна направлению тока. Решать задачи этой темы нужно так же как и в динамике или статике, т.е. расписав силы по осям координат или складывая силы по правилам сложения векторов.

Момент сил, действующих на рамку с током

Пусть рамка с током находится в магнитном поле, причём плоскость рамки перпендикулярна полю. Силы Ампера будут сжимать рамку, а их равнодействующая будет равна нулю. Если поменять направление тока, то силы Ампера поменяют своё направление, и рамка будет не сжиматься, а растягиваться. Если линии магнитной индукции лежат в плоскости рамки, то возникает вращательный момент сил Ампера. Вращательный момент сил Ампера равен:

где: S — площадь рамки, α — угол между нормалью к рамке и вектором магнитной индукции (нормаль — вектор, перпендикулярный плоскости рамки),

N – количество витков, B – индукция магнитного поля, I – сила тока в рамке.

 

Сила Лоренца

К оглавлению…

Сила Ампера, действующая на отрезок проводника длиной Δl с силой тока I, находящийся в магнитном поле B может быть выражена через силы, действующие на отдельные носители заряда. Эти силы называют силами Лоренца. Сила Лоренца, действующая на частицу с зарядом q в магнитном поле B, двигающуюся со скоростью v, вычисляется по следующей формуле:

Угол

α в этом выражении равен углу между скоростью и вектором магнитной индукции. Направление силы Лоренца, действующей на положительно заряженную частицу, так же, как и направление силы Ампера, может быть найдено по правилу левой руки или по правилу буравчика (как и сила Ампера). Вектор магнитной индукции нужно мысленно воткнуть в ладонь левой руки, четыре сомкнутых пальца направить по скорости движения заряженной частицы, а отогнутый большой палец покажет направление силы Лоренца. Если частица имеет отрицательный заряд, то направление силы Лоренца, найденное по правилу левой руки, надо будет заменить на противоположное.

Сила Лоренца направлена перпендикулярно векторам скорости и индукции магнитного поля. При движении заряженной частицы в магнитном поле

сила Лоренца работы не совершает. Поэтому модуль вектора скорости при движении частицы не изменяется. Если заряженная частица движется в однородном магнитном поле под действием силы Лоренца, а ее скорость лежит в плоскости, перпендикулярной вектору индукции магнитного поля, то частица будет двигаться по окружности, радиус которой можно вычислить по следующей формуле:

Сила Лоренца в этом случае играет роль центростремительной силы. Период обращения частицы в однородном магнитном поле равен:

Последнее выражение показывает, что для заряженных частиц заданной массы

m период обращения (а значит и частота, и угловая скорость) не зависит от скорости (следовательно, и от кинетической энергии) и радиуса траектории R.

 

Теория о магнитном поле

К оглавлению…

Магнитное взаимодействие токов

Если по двум параллельным проводам идёт ток в одном направлении, то они притягиваются; если в противоположных направлениях, то отталкиваются. Закономерности этого явления были экспериментально установлены Ампером.  Взаимодействие токов вызывается их магнитными полями: магнитное поле одного тока действует силой Ампера на другой ток и наоборот. Опыты показали, что модуль силы, действующей на отрезок длиной Δ

l каждого из проводников, прямо пропорционален силам тока I1 и I2 в проводниках, длине отрезка Δl и обратно пропорционален расстоянию R между ними:

где: μ0 – постоянная величина, которую называют магнитной постоянной. Введение магнитной постоянной в СИ упрощает запись ряда формул. Ее численное значение равно:

μ0 = 4π·10–7 H/A2 ≈ 1,26·10–6 H/A2.

Сравнивая приведенное только что выражение для силы взаимодействия двух проводников с током и выражение для силы Ампера нетрудно получить выражение для индукции магнитного поля создаваемого каждым из прямолинейных проводников с током на расстоянии R от него:

где: μ – магнитная проницаемость вещества (об этом чуть ниже). Если ток протекает по круговому витку, то в центре витка индукция магнитного поля определяется по формуле:

Силовыми линиями магнитного поля называют линии, по касательным к которым располагаются магнитные стрелки. Магнитной стрелкой называют длинный и тонкий магнит, его полюса точечны. Подвешенная на нити магнитная стрелка всегда поворачивается в одну сторону. При этом один её конец направлен в сторону севера, второй — на юг. Отсюда название полюсов: северный (N) и южный (S). Магниты всегда имеют два полюса: северный (обозначается синим цветом или буквой N) и южный (красным цветом или буквой S). Магниты взаимодействуют так же, как и заряды: одноименные полюса отталкиваются, а разноименные – притягиваются. Невозможно получить магнит с одним полюсом. Даже если магнит разломать, то у каждой части будет по два разных полюса.

Вектор магнитной индукции

Вектор магнитной индукции — векторная физическая величина, являющаяся характеристикой магнитного поля, численно равная силе, действующей на элемент тока в 1 А и длиной 1 м, если направление силовой линии перпендикулярно проводнику. Обозначается В, единица измерения — 1 Тесла. 1 Тл — очень большая величина, поэтому в реальных магнитных полях магнитную индукцию измеряют в мТл.

Вектор магнитной индукции направлен по касательной к силовым линиям, т.е. совпадает с направлением северного полюса магнитной стрелки, помещённой в данное магнитное поле. Направление вектора магнитной индукции не совпадает с направлением силы, действующей на проводник, поэтому силовые линии магнитного поля, строго говоря, силовыми не являются.

Силовая линия магнитного поля постоянных магнитов направлена по отношению к самим магнитам так, как показано на рисунке:

В случае магнитного поля электрического тока для определения направления силовых линий используют правило «Правой руки»: если взять проводник в правую руку так, чтобы большой палец был направлен по току, то четыре пальца, обхватывающие проводник, показывают направление силовых линий вокруг проводника:

В случае прямого тока линии магнитной индукции — окружности, плоскости которых перпендикулярны току. Вектора магнитной индукции направлены по касательной к окружности.

Соленоид — намотанный на цилиндрическую поверхность проводник, по которому течёт электрический ток I. Магнитное поле соленоида подобно полю прямого постоянного магнита. Внутри соленоида длиной l и количеством витков N создается однородное магнитное поле с индукцией (его направление также определяется правилом правой руки):

Линии магнитного поля имеют вид замкнутых линий — это общее свойство всех магнитных линий. Такое поле называют вихревым. В случае постоянных магнитов линии не оканчиваются на поверхности, а проникают внутрь магнита и замыкаются внутри. Это различие электрического и магнитного полей объясняется тем, что, в отличие от электрических, магнитных зарядов не существует.

Магнитные свойства вещества

Все вещества обладают магнитными свойствами. Магнитные свойства вещества характеризуются относительной магнитной проницаемостью μ, для которой верно следующее:

Данная формула выражает соответствие вектора магнитной индукции поля в вакууме и в данной среде. В отличие от электрического, при магнитном взаимодействии в среде можно наблюдать и усиление, и ослабление взаимодействия по сравнению с вакуумом, у которого магнитная проницаемость μ = 1. У диамагнетиков магнитная проницаемость μ немного меньше единицы. Примеры: вода, азот, серебро, медь, золото. Эти вещества несколько ослабляют магнитное поле. Парамагнетики — кислород, платина, магний — несколько усиливают поле, имея μ немного больше единицы. У ферромагнетиков — железо, никель, кобальт — μ >> 1. Например, у железа μ ≈ 25000.

 

Магнитный поток. Электромагнитная индукция

К оглавлению…

Явление электромагнитной индукции было открыто выдающимся английским физиком М.Фарадеем в 1831 году. Оно заключается в возникновении электрического тока в замкнутом проводящем контуре при изменении во времени магнитного потока, пронизывающего контур. Магнитным потоком Φ через площадь S контура называют величину:

где: B – модуль вектора магнитной индукции, α – угол между вектором магнитной индукции B и нормалью (перпендикуляром) к плоскости контура, S – площадь контура, N – количество витком в контуре. Единица магнитного потока в системе СИ называется Вебером (Вб).

Фарадей экспериментально установил, что при изменении магнитного потока в проводящем контуре возникает ЭДС индукции εинд, равная скорости изменения магнитного потока через поверхность, ограниченную контуром, взятой со знаком минус:

Изменение магнитного потока, пронизывающего замкнутый контур, может происходить по двум возможным причинам.

  1. Магнитный поток изменяется вследствие перемещения контура или его частей в постоянном во времени магнитном поле. Это случай, когда проводники, а вместе с ними и свободные носители заряда, движутся в магнитном поле. Возникновение ЭДС индукции объясняется действием силы Лоренца на свободные заряды в движущихся проводниках. Сила Лоренца играет в этом случае роль сторонней силы.
  2. Вторая причина изменения магнитного потока, пронизывающего контур, – изменение во времени магнитного поля при неподвижном контуре.

При решении задач важно сразу определить за счет чего меняется магнитный поток. Возможно три варианта:

  1. Меняется магнитное поле.
  2. Меняется площадь контура.
  3. Меняется ориентация рамки относительно поля.

При этом при решении задач обычно считают ЭДС по модулю. Обратим внимание также внимание на один частный случай, в котором происходит явление электромагнитной индукции. Итак, максимальное значение ЭДС индукции в контуре состоящем из N витков, площадью S, вращающемся с угловой скоростью ω в магнитном поле с индукцией В:

 

Движение проводника в магнитном поле

К оглавлению…

При движении проводника длиной l в магнитном поле B со скоростью v на его концах возникает разность потенциалов, вызванная действием силы Лоренца на свободные электроны в проводнике. Эту разность потенциалов (строго говоря, ЭДС) находят по формуле:

где: α — угол, который измеряется между направлением скорости и вектора магнитной индукции. В неподвижных частях контура ЭДС не возникает.

Если стержень длиной L вращается в магнитном поле В вокруг одного из своих концов с угловой скоростью ω, то на его концах возникнет разность потенциалов (ЭДС), которую можно рассчитать по формуле:

 

Индуктивность. Самоиндукция. Энергия магнитного поля

К оглавлению…

Самоиндукция является важным частным случаем электромагнитной индукции, когда изменяющийся магнитный поток, вызывающий ЭДС индукции, создается током в самом контуре. Если ток в рассматриваемом контуре по каким-то причинам изменяется, то изменяется и магнитное поле этого тока, а, следовательно, и собственный магнитный поток, пронизывающий контур. В контуре возникает ЭДС самоиндукции, которая согласно правилу Ленца препятствует изменению тока в контуре. Собственный магнитный поток Φ, пронизывающий контур или катушку с током, пропорционален силе тока I:

Коэффициент пропорциональности L в этой формуле называется коэффициентом самоиндукции или индуктивностью катушки. Единица индуктивности в СИ называется Генри (Гн).

Запомните: индуктивность контура не зависит ни от магнитного потока, ни от силы тока в нем, а определяется только формой и размерами контура, а также свойствами окружающей среды. Поэтому при изменении силы тока в контуре индуктивность остается неизменной. Индуктивность катушки можно рассчитать по формуле:

где: n — концентрация витков на единицу длины катушки:

ЭДС самоиндукции, возникающая в катушке с постоянным значением индуктивности, согласно формуле Фарадея равна:

Итак ЭДС самоиндукции прямо пропорциональна индуктивности катушки и скорости изменения силы тока в ней.

Магнитное поле обладает энергией. Подобно тому, как в заряженном конденсаторе имеется запас электрической энергии, в катушке, по виткам которой протекает ток, имеется запас магнитной энергии. Энергия Wм магнитного поля катушки с индуктивностью L, создаваемого током I, может быть рассчитана по одной из формул (они следуют друг из друга с учётом формулы Φ = LI):

Соотнеся формулу для энергии магнитного поля катушки с её геометрическими размерами можно получить формулу для объемной плотности энергии магнитного поля (или энергии единицы объёма):

 

Правило Ленца

К оглавлению. ..

Инерция – явление, происходящее и в механике (при разгоне автомобиля мы отклоняемся назад, противодействуя увеличению скорости, а при торможении отклоняемся вперёд, противодействуя уменьшению скорости), и в молекулярной физике (при нагревании жидкости увеличивается скорость испарения, самые быстрые молекулы покидают жидкость, уменьшая скорость нагревания) и так далее. В электромагнетизме инерция проявляется в противодействии изменению магнитного потока, пронизывающего контур. Если магнитный поток нарастает, то возникающий в контуре индукционный ток направлен так, чтобы препятствовать нарастанию магнитного потока, а если магнитный поток убывает, то возникающий в контуре индукционный ток направлен так, чтобы препятствовать убыванию магнитного потока.

Правило Ленца для определения направления индукционного тока: возникающий в контуре индукционный ток имеет такое направление, что создаваемое им магнитное поле препятствует изменению магнитного потока, которое вызывало этот ток.

Глава 22. Магнитные взаимодействия. Магнитная индукция.Силы Лоренца и Ампера

Если заряд движется, то наряду с электрическим полем он создает еще одно поле — магнитное, которое действует на другие движущиеся заряды. В результате возникает дополнительное (наряду с кулоновским) взаимодействие движущихся электрических зарядов, которое называется магнитным. В результате магнитного взаимодействия возникает взаимодействие проводников с током.

В 1820 г. датский физик Х. Эрстед обнаружил, что проводник с током действует на магнитную стрелку. После этого стало ясно, что магнитное взаимодействие движущихся электрических зарядов и токов и взаимодействие постоянных магнитов имеют одну и ту же природу. На основании подробных исследований А. Ампер установил, что взаимодействие постоянных магнитов между собой и с токами можно объяснить, если предположить, что внутри магнитов есть электрические токи (в настоящее время известно, что эти токи имеют внутримолекулярную природу).

Для характеристики магнитного поля вводится векторная величина, которая называется индукцией магнитного поля и которая позволяет найти силы, действующие со стороны магнитного поля на движущиеся заряды. Как правило, эту величину обозначают буквой . Для нахождения индукции в каждой точке магнитного поля, созданного проводником с током, используется закон Био-Савара-Лапласа и принцип суперпозиции. Закон Био-Савара-Лапласа позволяет найти поле , созданное бесконечно малым элементом проводника, а принцип суперпозиции требует сложить векторы индукции, созданные всеми элементами проводников. Закон Био-Савара-Лапласа в школьный курс физики, однако, не входит. В задачи ЕГЭ входят только вопросы, связанные с направлением вектора магнитной индукции (но не с величиной). Существует несколько вариантов правила нахождения направления вектора . Наиболее удобным является правило буравчика — оно более универсально, чем правило левой руки. Правило буравчика утверждает, что если вкручивать правыйбуравчик1 по току в проводнике, то направление движения ручки в каждой точке пространства покажет направление вектора индукции магнитного поля в этой точке. Относительно величины достаточно помнить, что чем дальше от проводника, тем меньше индукция, и что внутри бесконечной катушки (бесконечного соленоида) магнитное поле направлено вдоль оси катушки и однородно.

Магнитное поле можно изобразить графически с помощью линий магнитной индукции. Линии магнитной индукции — воображаемые линии, касательные к которым в каждой точке совпадают с направлением вектора в этой точке. Линии магнитной индукции проводят так, что их густота в каждой области пространства пропорциональна величине индукции в этой области. В отличие от силовых линий электрического поля линии магнитной индукции всегда являются замкнутыми.

На электрический заряд величиной , движущийся со скоростью в магнитном поле с индукцией , со стороны магнитного поля действует сила, которая называется силой Лоренца

(22.1)

где — угол между скоростью и вектором индукции. Направление силы Лоренца определяется следующим образом (см. рисунок).

1. Сила Лоренца перпендикулярна плоскости, в которой лежат векторы скорости заряда и индукции магнитного поля (на рисунке эта плоскость показана тонким пунктиром).

2. Выбор между двумя перпендикулярными направлениями осуществляется с помощью правила буравчика (или правила левой руки): если вращать правый буравчик так, что его ручка движется от вектора к вектору , то направление его вкручивания указывает направление силы Лоренца, действующей на положительный заряд (траектория ручки буравчика показана на рисунке изогнутой стрелкой).

3. Для отрицательного заряда направление силы Лоренца противоположно.

Можно также определять направление силы Лоренца по правилу левой руки: левую руку нужно расположить так, чтобы вектор входил в ладонь, направление четырех пальцев совпадало с направлением вектора скорости заряда, тогда направление отогнутого под прямым углом к четырем пальцам большого пальца покажет направление силы, действующей на положительный заряд (на отрицательный заряд действует сила противоположного направления).

Поскольку магнитное поле действует на движущиеся заряды, то магнитное поле действует и на проводник, по которому течет электрический ток. Если в магнитном поле с индукцией находится проводник длиной , по которому течет ток , то на этот проводник действует сила

(22.2)

где — угол между током и вектором индукции. Направлен вектор силы (22.2) перпендикулярно плоскости, в которой лежат вектор и проводник, причем в таком направлении, что если поставить правый буравчик перпендикулярно указанной плоскости и вращать его так, что ручка вращается от тока к вектору , то направление его вкручивания покажет направление силы (см. рисунок; плоскость в которой лежат проводник и вектор индукции обозначена тонким пунктиром, движение ручки буравчика — изогнутой стрелкой). Также для нахождения направления силы можно использовать правило левой руки. Сила (22.2), действующая со стороны магнитного поля на проводник с током, называется силой Ампера.

Рассмотрим теперь задачи.

Правильный ответ в задаче 22.1.14 (магнитное поле создается движущимися заряженными телами), в задаче 22.1.22 (в магнитном веществе есть незатухающие электрические токи). Что же касается того, заряжен магнит или нет, то от этого существование магнитного поля (если магнит покоится) не зависит.

В задаче 22.1.3 следует воспользоваться правилом буравчика. Если вкручивать буравчик по направлению тока в проводнике, то в точке его ручка будет двигаться за чертеж. Следовательно, за чертеж направлен в точке и вектор индукции магнитного поля (ответ 1).

Если вкручивать буравчик по току в кольце (в любой точке кольца), то ручка буравчика в центре кольца будет двигаться за чертеж. Поэтому правильный ответ в задаче 22.1.43.

Поскольку угол между скоростью заряда и вектором магнитной индукции равен нулю (задача 22.1.5), то согласно формуле (22. 1) сила Лоренца, действующая на этот заряд, равна нулю (ответ 4).

Применим к проводнику с током из задачи 22.1.6 формулу (22.2) для силы Ампера. Имеем (ответ 2).

Как следует из формулы (22.2) сила Ампера равна нулю, если угол между током и индукцией равен нулю или 180°. Из приведенных на рисунке в задаче 22.1.7 проводников, таковым является только проводник 1. Поэтому на него магнитное поле не действует (ответ 1).

Применяем к частице из задачи 22.1.8 (см. рисунок) правила нахождения направления силы Лоренца (пункты 1-3 после формулы (22.1)). Во-первых, сила Лоренца направлена перпендикулярно плоскости, в которой лежат векторы скорости заряда и индукции магнитного поля — т.е. либо за чертеж, либо на нас. Во-вторых, при вращении ручки буравчика, поставленного на чертеж в ту точку, где находится заряд, от вектора к вектору (в направлении меньшего угла между ними), буравчик будет «выкручиваться» из чертежа. А по-скольку частица заряжена положительно, сила Лоренца направлена «на нас» (ответ 1).

Используя правила для силы Ампера (формула (22.2) и текст после нее), найдем, что сила Ампера, действующая со стороны магнитного поля на проводник с током в задаче 22.1.9, направлена «от нас» (ответ 3).

В задаче 22.1.10 следует сначала найти направление вектора магнитной индукции поля провода в той точке, где находится заряд, а затем использовать правила для силы Лоренца (формула (22.1) и текст за ней). Согласно результатам задачи 22.1.3, вектор в той точке, где находится заряд, направлен за чертеж (см. рисунок).

Вектор силы Лоренца направлен перпендикулярно плоскости, в которой лежат векторы и , т.е. либо к проводу, либо от него. Ставим буравчик перпендикулярно этой плоскости и вращаем его так, что ручка движется от вектора к вектору (см. рисунок; буравчик нужно вращать по часовой стрелке, если смотреть снизу). При таком вращении буравчик будет вкручиваться вверх. А поскольку электрон заряжен отрицательно, то действующая на него сила направлена противоположно, т.е. от провода (ответ 2).

В задаче 22.2.1 используем принцип суперпозиции. Ток в горизонтальном кольце создает поле в его центре с индукцией, направленной вверх, ток в вертикальном кольце — с индукцией, направленной вправо (см. задачу 22.1.4.). Результат сложения этих векторов — индукция суммарного магнитного поля — направлена на «северо-восток» (ответ 1).

Ток в верхнем проводе (задача 22.2.2) создает поле с индукцией, направленной «за чертеж», ток в нижнем — «на нас». Результат их сложения зависит от величин этих векторов. Поскольку поле нижнего провода в точке больше поля верхнего (меньше расстояние), то вектор суммы направлен «на нас» (ответ 1).

Сила Лоренца в любой момент времени перпендикулярна скорости частицы. Поэтому угол между бесконечно малым перемещением частицы в любой момент времени и силой Лоренца, действующей на частицу в этот момент времени, — прямой. А поскольку в формулу для работы силы на бесконечно малом участке перемещения входит косинус угла между силой и перемещением, то работа силы Лоренца равна нулю (задача 22.2.3 — ответ 3). Из этих рассуждений и теоремы об изменении кинетической энергии следует, что заряженная частица, движущаяся под действием магнитного поля, изменяет направление, но не величину своей скорости.

Если заряженная частица влетает в однородное магнитное поле перпендикулярно линиям магнитной индукции, то она движется по окружности, причем эта окружность лежит в плоскости, перпендикулярной линиям магнитной индукции. Радиус окружности можно найти из второго закона Ньютона для этой частицы

(22.3)

где и — масса частицы и ее заряд, — ускорение, — скорость, которая не изменяется по величине (см. предыдущую задачу), — индукция магнитного поля. В формуле (22.3) использовано известное выражение для центростремительного ускорения . Из формулы (22.3) получаем для радиуса окружности

(22.4)

Применяя формулу (22.4) к задаче 22.2.4 находим отношение радиусов окружности первой и второй частиц

(ответ 2).

Найдем сначала скорости протона и -частицы, ускоренных одним и тем же напряжением (задача 22.2.5). По теореме об изменении кинетической энергии имеем

где и — масса частицы и ее заряд, — скорость, которую частица приобретает после разгона (здесь предполагается, что начальная скорость частицы равна нулю). Из этой формулы находим отношение скоростей протона и -частицы , ускоренных одним и тем же напряжением

Поскольку заряд протона вдвое меньше заряда -частицы, а масса вчетверо меньше, то . Теперь из формулы (22.4) находим отношение радиусов окружности протона и  -частицы, ускоренных одним и тем же электрическим напряжением и движущихся в одном и том же магнитном поле

(ответ 4).

Период обращения заряженной частицы в магнитном поле (задача 22.2.6) можно найти из следующих соображений. В однородном магнитном поле частица движется по окружности и за период проходит путь, равный длине этой окружности , где — ее радиус. Используя формулу (22.4) для радиуса траектории, получим для периода обращения

где — скорость частицы, — ее масса, — заряд, — индукция магнитного поля. Отсюда заключаем, что период обращения заряженной частицы в магнитном поле не зависит от ее скорости (ответ 3).

Индукция магнитного поля в задаче 22.2. 7 должна быть направлена так, чтобы сила Лоренца, действующая на электрон, была направлена к центру окружности, по которой он движется (см. рисунок). А поскольку сила Лоренца перпендикулярна скорости и индукции, то вектор индукции может быть направлен в этой ситуации только «за чертеж» или «на нас». Воспользуемся далее правилом буравчика (см. текст после формулы (22.1)): если вращать буравчик так, что его ручка будет вращаться от скорости заряда к индукции магнитного поля , то направление его вкручивания указывает направление силы Лоренца, действующей на положительный заряд. Для электрона ( < 0) направление силы противоположно. Непосредственной проверкой убеждаемся, что вектор индукции направлен «за чертеж» (ответ 4).

В области среднего провода (задача 22.2.8) ток в верхнем проводе создает магнитное поле с индукцией, направленной «от нас», ток в нижнем — «на нас» (см. задачу 22.1.3). Но ток в нижнем проводе вдвое меньше тока в верхнем, а индукция поля — пропорциональна току. Поэтому индукция суммарного поля верхнего и нижнего проводов в области среднего провода направлена «от нас». Согласно правилам нахождения направления силы Ампера (см. текст после формулы (22.2)) находим, что сила, действующая на средний провод со стороны магнитного поля верхнего и нижнего проводов, направлена вверх (ответ 1). Отметим, что из приведенных рассуждений также следует, что два параллельных провода, по которым текут токи одинакового направления притягиваются, противоположного — отталкиваются.

В задаче 22.2.9 магнитное поле действует на рамку следующим образом. На стороны и , которые параллельны линиям индукции, поле не действует. На стороны и действуют силы Ампера, равные по величине , где — ток в рамке, — индукция магнитного поля, — длина стороны. Сила, действующая на сторону , направлена «на нас», на сторону — «от нас». Поскольку суммарная сила, действующая на рамку, равна нулю, как целое рамка перемещаться в пространстве не будет, а будет вращаться вокруг оси, показанной на рисунке пунктиром (ответ 4).

Задача 22.2.10 по формуле (22.2) находим силы Ампера, действующие на стороны треугольника

где — ток в контуре, и — длины сторон и , — индукция магнитного поля (последняя из приведенных формул следует из того, что сторона параллельна линиям индукции). Из теоремы синусов для треугольника

заключаем, что , а из правил для направления силы Ампера — что один из векторов или направлен «за чертеж», один — «на нас» (в зависимости от направления тока в контуре). Поэтому правильный ответ в задаче — 3.

формула, от чего зависит, линии индукции

 

Все мы знаем, что есть магниты более сильные и менее сильные. Маленький магнитик сможет притянуть пару гвоздей и все, а гораздо более мощный электромагнит домофона удерживает дверь в подъезд так, что несколько взрослых мужчин не смогут открыть ее силой.

Величина, характеризующая величину силы магнита

То есть, мы можем говорить о некой величине, характеризующей величину силы магнитов, а точнее, магнитного поля, создаваемого ими. Магнитное поле характеризуется векторной величиной, которая носит название индукции магнитного поля или магнитной индукции. (см. подробнее электромагнитная индукция)

Обозначается индукция буквой B. Магнитная индукция это не сила, действующая на проводники, это величина, которая находится через данную силу по следующей формуле:

B=F / (I*l)

Или в виде определения:

Модуль вектора магнитной индукции B равен отношению модуля силы F, с которой магнитное поле действует на расположенный перпендикулярно магнитным линиям проводник с током, к силе тока в проводнике I и длине проводника l.

От чего зависит магнитная индукция

Магнитная индукция не зависит ни от силы тока, ни от длины проводника, она зависит только от магнитного поля. То есть, если мы, например, уменьшим силу тока в проводнике, не меняя больше ничего, то уменьшится не индукция, с которой сила тока связана прямо пропорционально, а сила воздействия магнитного поля на проводник. Величина же индукции останется постоянной. В связи с этим индукцию можно считать количественной характеристикой магнитного поля.

Измеряется магнитная индукция в теслах (1 Тл). При этом 1 Тл=1 Н/(А*м) .

Линии индукции магнитного поля

Магнитная индукция имеет направление. Графически ее можно зарисовывать в виде линий. Линии индукции магнитного поля это и есть то, что мы до сих пор в более ранних темах называли магнитными линиями или линиями магнитного поля. Так как мы выше вывели определение магнитной индукции, то мы можем дать определение и линиям магнитной индукции:

Линии магнитной индукции это линии, касательные к которым в каждой точке поля совпадают с направлением вектора магнитной индукции.

В однородном магнитном поле линии магнитной индукции параллельны, и вектор магнитной индукции будет направлен так же во всех точках.

В случае неоднородного магнитного поля, например, поля вокруг проводника с током, вектор магнитной индукции будет меняться в каждой точке пространства вокруг проводника, а касательные к этому вектору создадут концентрические окружности вокруг проводника. Так и будут выглядеть линии индукции магнитного поля расширяющиеся окружности вокруг проводника.

Нужна помощь в учебе?



Предыдущая тема: Обнаружение магнитного поля по его действию на электрический ток
Следующая тема:&nbsp&nbsp&nbspМагнитный поток: определение, направление и количество + пример

основные формулы, законы и правила

Магнитное поле — это форма материи, окружающей движущиеся электрические заряды. Магнитное поле окружает проводники с током.

Силовой характеристикой магнитного поля является магнитная индукция.

Магнитная индукция

— это величина, равная отношению максимального момента силы, вращающей контур с током в магнитном поле, к силе тока в этом контуре и его площади:

Другое определение магнитной индукции: магнитная индукция — это величина, равная отношению максимальной силы, действующей на проводник с током в магнитном поле, к силе тока в нем и длине этого проводника в магнитном поле:

Магнитная индукция — векторная величина. Вектор магнитной индукции совпадает по направлению с положительной нормалью

к плоскости контура. За

направление положительной нормали п принято направление поступательного движения правого винта (буравчика), когда его головка вращается по току в контуре (рис. 208).

Правым винтом может служить ваша правая рука. Если свернуть четыре пальца правой руки в направлении тока в контуре, то большой палец, отставленный на 90°, покажет направление положительной нормали и вектора магнитной индукции.

Единица магнитной индукции в СИ — тесла (Тл).

Магнитное поле изображают графически с помощью магнитных силовых линий или линий вектора магнитной индукции.

В природе не существует магнитных зарядов, поэтому линии вектора магнитной индукции всегда замкнуты.

Магнитное поле является вихревым, в отличие от потенциального электростатического поля, линии которого всегда разомкнуты, т. к. начинаются и оканчиваются на электрических зарядах. Линии вектора магнитной индукции охватывают проводники с током. Линии вектора магнитной индукции поля прямого тока представляют собой концентрические окружности с центром на проводнике с током (рис. 209). Их направление можно определить с помощью правого винта (или с помощью вашей правой руки: если большой палец правой руки направить по направлению тока в проводнике, то четыре загнутых пальца покажут направление линии магнитной индукции). По мере удаления от проводника с током индукция магнитного поля этого тока уменьшается.

Магнитное поле, в каждой точке которого вектор магнитной индукции одинаков, называется однородным. Линии магнитной индукции однородного поля представляют собой прямые, расположенные на одинаковом расстоянии друг от друга. Чем гуще они располагаются, тем больше магнитная индукция.

Примером однородного магнитного поля является магнитное поле внутри длинного соленоида — катушки с током (рис. 210).

Линии магнитной индукции выходят из северного полюса N и входят в его южный полюс S.

Магнитное поле полосового магнита (рис. 211) наибольшее на его полюсах, а в центре его магнитная индукция равна нулю.

Если в однородное поле внести контур с током, расположив его плоскость параллельно линиям магнитной индукции, то на стороны контура, перпендикулярным линиям магнитной индукции, будет действовать пара сил Ампера, которая создаст максимальный вращающий момент сил

, равный произведению индукции магнитного поля, силы тока в ней и ее площади:

Если плоскость контура расположена под углом к линиям вектора индукции однородного магнитного поля, то момент сил определяет формула

Здесь

— угол между вектором индукции магнитного поля и нормалью к плоскости рамки.

Момент сил, вращающих контур с током в однородном магнитном поле, равен произведению индукции этого поля, силы тока в контуре, площади контура и синуса угла между векторами магнитной индукции и нормали к плоскости контура.

Если плоскость контура перпендикулярна линиям вектора магнитной индукции, то вращающий момент сил равен 0, а силы Ампера действуют в плоскости контура, деформируя его.

Направление силы Ампера можно определить по правилу левой руки: если ладонь левой руки расположить так, чтобы магнитные линии входили в ладонь, а четыре вытянутых пальца направить по току в проводнике, то большой палец, отставленный на 90 , покажет направление силы Ампера, действующей на этот проводник в данном магнитном поле (рис. 212).

Если проводник с током расположить параллельно магнитным линиям, то сила Ампера на него действовать не будет.

Сила Ампера

Величину силы Ампера определяет закон Ампера: сила F, действующая на проводник с током в однородном магнитном поле, равна произведению магнитной индукции этого поля В, силы тока в проводнике I, длины проводника в магнитном поле I и синуса угла а между направлением магнитного поля и направлением тока в проводнике:

Сила, с которой магнитное поле действует на движущийся в нем заряд, называется силой Лоренца.

Сила Лоренца

Сила Лоренца

действующая на заряд q, движущийся в однородном магнитном поле, равна произведению индукции этого поля В на заряд, на скорость его движения v и на синус угла между направлением магнитного поля и направлением движения заряда

Определить направление силы Лоренца можно тоже по правилу левой руки: если ладонь левой руки расположить так, чтобы магнитные линии входили в нее, а четыре вытянутых пальца направить по направлению движения положительного заряда (или против направления движения отрицательного заряда), то большой палец, отставленный на 90″, покажет направление силы Лоренца.

Заряженная частица, влетевшая в однородное магнитное поле перпендикулярно его магнитным линиям, движется равномерно по окружности, охватывающей магнитные линии.

При этом сила Лоренца направлена по радиусу к центру окружности.

На рис. 213 положительно заряженная частица с зарядом q, влетевшая в направлении, показанном вектором v в однородное магнитное поле индукцией В, направленном за чертеж, движется вокруг магнитных линий против часовой стрелки.

Если заряженная частица влетает в магнитное поле под углом к магнитным линиям, то она станет двигаться по винтовой линии (рис. 214), вращаясь по окружности с линейной скоростью, равной нормальной составляющей

вектора скорости usin а, и одновременно перемещаясь

равномерно вдоль линий вектора индукции магнитного поля с тангенциальной составляющей vcos а вектора скорости

.

Расстояние х, которое она пролетит вдоль магнитной линии за один оборот, называется шагом винта. Поскольку вдоль магнитной линии частица движется с постоянной скоростью и, то шаг винта равен

Здесь Т — период, т. е. время одного оборота частицы вокруг магнитных линий.

Сила Лоренца всегда перпендикулярна вектору скорости и, следовательно, вектору перемещения заряда, поэтому она работы перемещения заряда в магнитном поле не совершает, вследствие чего кинетическая энергия заряда, движущегося в магнитном поле под действием силы Лоренца, не изменяется.

Если заряженная частица движется одновременно в электрическом и магнитном полях (т.е. в электромагнитном поле), то на нее действует обобщенная сила Лоренца, равная векторной сумме силы Лоренца, действующей на нее со стороны магнитного поля, и силы Кулона, действующей со стороны электрического поля.

Пусть в однородном магнитном поле индукцией находится некоторая площадка S (рис. 215).

Магнитный поток Ф, создаваемый однородным магнитным полем сквозь некоторую площадку, равен произведению индукции этого магнитного поля В на величину площадки S и на косинус угла а между вектором магнитной индукции и нормалью

к площадке.

Если площадка S расположена перпендикулярно магнитным линиям однородного поля, то магнитный поток, пересекающий ее, максимален:

Если площадка S расположена параллельно магнитным линиям, то они ее не пересекают, поэтому магнитный поток через площадку в этом случае равен нулю.

Магнитный поток

Магнитный поток — скалярная алгебраическая величина, т.е. он может быть положителен и отрицателен, поскольку косинус угла может быть больше и меньше нуля.

Если магнитный поток пересекает замкнутую поверхность (представьте ее в виде надутого воздушного шарика), то, поскольку все магнитные линии непрерывны и замыкаются сами на себя, число входящих в эту поверхность магнитных линий, создающих отрицательный поток, будет равно числу выходящих магнитных линий, создающих численно такой же по модулю, но положительный поток. Поэтому полный поток вектора магнитной индукции сквозь замкнутую поверхность равен нулю. Это важное свойство магнитного поля свидетельствует об отсутствии в природе магнитных зарядов и вихревом характере магнитного поля.

Единица магнитного потока в СИ — вебер (Вб).

Когда магнитный поток сквозь площадь, ограниченную проводящим контуром, изменяется, в этом контуре возникает индукционный ток.

Правило Ленца: индукционный ток всегда направлен так, что своим магнитным полем он противодействует изменению магнитного потока, вызвавшего этот ток.

Обратимся к рис. 216, а). Когда магнитный поток сквозь контур, создаваемый внешним по отношению к контуру магнитным полем индукцией

, нарастает , индукционный ток в контуре направлен так, что его магнитное поле индукцией (на рис. 216, а) оно изображено штриховыми стрелками), антинаправлено внешнему магнитному полю, противодействуя увеличению магнитного потока. Отметим, что направление тока связано с направлением своего магнитного поля правилом правого винта — буравчика. Когда же магнитный поток, создаваемый внешним магнитным полем индукцией убывает (рис. 216, б), индукционный ток в контуре изменяет свое направление на противоположное и при этом его магнитное поле оказывается сонаправленным с внешним полем . Теперь магнитное поле индукционного тока противодействует убыли магнитного потока, создаваемого внешним магнитным полем сквозь контур, поддерживая его.

Явление возникновения индукционного тока в контуре при изменении магнитного потока, пересекающего этот контур, называется электромагнитной индукцией. По закону Ома сила индукционного тока

прямо пропорциональна ЭДС индукции и обратно пропорциональна сопротивлению контура В:

Закон Фарадея

Закон Фарадея для электромагнитной индукции: ЭДС электромагнитной индукции, возникающая в контуре при всяком изменении магнитного потока, пересекающего этот контур, равна скорости изменения магнитного потока, взятой со знаком минус,

Здесь

— ЭДС индукции в контуре, — скорость изменения магнитного потока, пересекающего контур, N — число витков в контуре (безразмерное).

Эта формула справедлива, когда магнитный поток изменяется монотонно, т.е. когда за равные промежутки времени

он изменяется на одинаковую величину и ЭДС индукции постоянна. Если же магнитный поток изменяется произвольно, то увеличиваясь, то уменьшаясь, что бывает при вращении контура в магнитном поле, то пользоваться этой формулой для определения мгновенного значения ЭДС индукции нельзя, по ней можно определить только среднее значение ЭДС индукции.

При произвольном изменении магнитного потока сквозь контур ЭДС индукции равна первой производной магнитного потока по времени, взятой со знаком минус:

Здесь

— первая производная магнитного потока по времени.

Знак минус в этих формулах объясняется правилом Ленца.

Если контур, пересекаемый переменным магнитным потоком, содержит не один, а N витков, то ЭДС индукции в нем будет в N раз больше, чем в одном витке. При этом предыдущие формулы примут вид:

ЭДС индукции, возникающая в проводнике, движущемся поступательно в однородном магнитном поле под углом к магнитным линиям, равна произведению индукции этого поля на скорость проводника, на его длину в этом поле и на синус угла между вектором индукции магнитного и вектором скорости проводника:

ЭДС индукции

, возникающая в контуре, вращающемся равномерно в однородном магнитном поле, равна произведению угловой скорости контура на индукцию В магнитного поля, на площадь контура S и на синус угла а между вектором магнитной индукции и нормалью к плоскости контура:

В случае, когда плоскость контура параллельна магнитным линиям, угол

. Тогда ЭДС индукции в контуре будет максимальна.

Если контур содержит N витков, то ЭДС индукции в нем в N раз больше, чем в одном витке:

Явление возникновения ЭДС индукции и индукционного тока в контуре вследствие изменения тока, текущего в этом контуре, называется явлением самоиндукции.

Магнитный поток Ф сквозь катушку (или контур любой иной формы) прямо пропорционален силе тока в ней, т.е. между этими величинами существует прямо пропорциональная зависимость:

Здесь L — коэффициент пропорциональности между током и связанным с ним магнитным потоком. Он называется коэффициентом самоиндукции контура или его индуктивностью. Величина индуктивности зависит от формы и размеров самого контура, а также от магнитных свойств среды, и постоянна для данного контура. Индуктивность контура — скалярная положительная величина. Она не зависит от наличия или отсутствия тока в нем. Индуктивность катушек заводского изготовления указывается в их паспорте.

Единица индуктивности в СИ — генри (Гн).

ЭДС самоиндукции

, возникающая в контуре при изменении тока в нем, прямо пропорциональна скорости изменения силы тока в контуре, взятой со знаком «минус»:

Здесь

—скорость изменения силы тока, т.е. изменение силы тока за единицу времени.

Если ток в контуре изменяется произвольно, то пользоваться этой формулой для определения мгновенной ЭДС самоиндукции нельзя, по ней можно определить лишь среднее значение ЭДС самоиндукции за время

. Для определения мгновенного значения ЭДС самоиндукции в этом случае надо пользоваться формулой

Мгновенная ЭДС самоиндукции прямо пропорциональна первой производной силы тока по времени, взятой со знаком «минус».

Магнитное поле, как и всякое силовое поле, обладает энергией.

Энергия магнитного поля катушки с током соленоида равна половине произведения индуктивности этого соленоида на квадрат силы тока в нем:

Поскольку магнитное поле размыто по пространству, то, чтобы охарактеризовать его энергетические свойства, вводят величину, равную энергии магнитного поля в единице объема пространства, занятого этим полем. Эта величина называется объемной плотностью энергии магнитного поля

.

Объемная плотность энергии магнитного поля

равна отношению энергии магнитного поля к объему V пространства, занятого им:

Объемная плотность энергии магнитного поля прямо пропорциональна квадрату магнитной индукции этого поля и обратно пропорциональна относительной магнитной проницаемости окружающей среды:

Эта теория со страницы подробного решения задач по физике, там расположена теория и подробное решения задач по всем темам физики:

Задачи по физике с решением

Возможно вам будут полезны эти страницы:

Формула напряженности магнитного поля в физике

Содержание:

Определение и формула напряженности магнитного поля

Определение

Напряженностью магнитного поля $\bar{H}$ называют векторную физическую величину, направленную по касательной к силовым линиям поля, являющуюся характеристикой магнитного поля, равную:

$$\bar{H}=\frac{\bar{B}}{\mu_{0}}-\bar{J}(1)$$

где $\bar{B}$ – вектор магнитной индукции, $\mu_{0}=4 \pi \cdot 10^{-7}$ Гн/м(Н/А2)- магнитная постоянная, $\bar{j}$ – вектор намагниченности среды в исследуемой точке поля.

Для магнитного поля в вакууме напряженность магнитного поля определяется выражением:

$$\bar{H}=\frac{\bar{B}}{\mu_{0}}$$

В изотропной среде формула (1) преобразуется к виду:

$$\bar{H}=\frac{\bar{B}}{\mu_{0} \mu}$$

где $\mu$ – скалярная величина, называемая относительной магнитной проницаемостью среды (или просто магнитной проницаемостью). В изотропной среде векторы напряженности магнитного поля и магнитной индукции совпадают по направлению.

Иногда напряженность магнитного поля $d \bar{H}$ определяют как векторную величину, направленную по касательной к силовой линии поля, по модулю равной отношению силы (dF), с которой поле воздействует на единичный элемент тока (dl), который расположен перпендикулярно полю в вакууме, к магнитной постоянной:

$$d H=\frac{d F}{\mu_{0} I d l}$$

Закон Био-Савара-Лапласа

Это важнейший в электромагнетизме закон. Он определяет вектор напряженности $d \bar{H}$ в произвольной точке магнитного поля, которое создает в вакууме элементарный проводник длинны dl с постоянным током I:

$$d \bar{H}=\frac{1}{4 \pi} \frac{I}{r^{3}} d \bar{l} \times \bar{r}(5)$$

где $d \bar{l}$ – вектор элемента проводника, который по модулю равен длине проводника, направление совпадает с направлением тока; $\bar{r}$ – радиус–вектор, который проводят от рассматриваемого элементарного проводника к точке рассмотрения поля; $r=|\bar{r}|$ .

Вектор $d \bar{H}$ – перпендикулярен плоскости, в которой находятся векторы $d \bar{l}$ и $\bar{r}$, и направлен так, что из его конца вращение вектора $d \bar{l}$ по кратчайшему пути до совмещения с вектором $\bar{r}$ происходило по часовой стрелке. Для нахождения направления вектора $d \bar{H}$ можно использовать правило буравчика (Буравчик (винт) вращаем так, чтобы его поступательное движение совпадало с направлением тока, тогда направление, по которому вращается ручка винта, совпадает с направлением вектора напряженности поля, которое создает рассматриваемый ток).

Закон Био-Савара-Лапласа дает возможность вычислять величину полной напряженности магнитного поля, которое создает ток, текущий по проводнику любой формы.

Для нахождения полной напряженности магнитного поля, которое создает в исследуемой точке ток I, который течет по проводнику l, следует векторно суммировать все элементарные напряженности $d \bar{H}$, порождаемые элементами проводника и найденные по формуле (4). {2}}$

Читать дальше: Формула напряженности электрического поля.

Магнитный поток, индукция и закон Фарадея

Индуцированные ЭДС и магнитный поток

Закон индукции Фарадея гласит, что электродвижущая сила индуцируется изменением магнитного потока.

Цели обучения

Объясните взаимосвязь между магнитным полем и электродвижущей силой

Основные выводы

Ключевые моменты
  • Это изменение потока магнитного поля, которое приводит к появлению электродвижущей силы (или напряжения).
  • Магнитный поток (часто обозначаемый Φ или Φ B ), проходящий через поверхность, является составляющей магнитного поля, проходящего через эту поверхность.
  • В самом общем виде магнитный поток определяется как [латекс] \ Phi _ {\ text {B}} = \ iint _ {\ text {A}} \ mathbf {\ text {B}} \ cdot \ text {d} \ mathbf {\ text {A}} [/ latex]. Это интеграл (сумма) всего магнитного поля, проходящего через бесконечно малые элементы площади dA.
Ключевые термины
  • векторная площадь : вектор, величина которого соответствует рассматриваемой области, а направление перпендикулярно площади поверхности.
  • гальванометр : аналоговое измерительное устройство, обозначенное буквой G, которое измеряет ток, используя отклонение стрелки, вызванное силой магнитного поля, действующей на провод с током.

Индуцированная ЭДС

Аппарат, использованный Фарадеем для демонстрации того, что магнитные поля могут создавать токи, показан на следующем рисунке. Когда переключатель замкнут, магнитное поле создается в катушке в верхней части железного кольца и передается (или направляется) в катушку в нижней части кольца.Гальванометр используется для обнаружения любого тока, наведенного в отдельной катушке внизу.

Аппарат Фарадея : Это аппарат Фарадея для демонстрации того, что магнитное поле может производить ток. Изменение поля, создаваемого верхней катушкой, вызывает ЭДС и, следовательно, ток в нижней катушке. Когда переключатель разомкнут и замкнут, гальванометр регистрирует токи в противоположных направлениях. Когда переключатель остается замкнутым или разомкнутым, через гальванометр не течет ток.

Было обнаружено, что каждый раз, когда переключатель замыкается, гальванометр обнаруживает ток в одном направлении в катушке внизу. Каждый раз при размыкании переключателя гальванометр обнаруживает ток в противоположном направлении. Интересно, что если переключатель остается замкнутым или разомкнутым в течение некоторого времени, через гальванометр нет тока. Замыкание и размыкание переключателя индуцирует ток. Это изменение магнитного поля, которое создает ток. Более важным, чем текущий ток, является вызывающая его электродвижущая сила (ЭДС).Ток является результатом ЭДС, индуцированной изменяющимся магнитным полем, независимо от того, есть ли путь для протекания тока.

Магнитный поток

Магнитный поток (часто обозначаемый Φ или Φ B ), проходящий через поверхность, является составляющей магнитного поля, проходящего через эту поверхность. Магнитный поток через некоторую поверхность пропорционален количеству силовых линий, проходящих через эту поверхность. Магнитный поток, проходящий через поверхность с векторной площадью A, равен

[латекс] \ Phi_ \ text {B} = \ mathbf {\ text {B}} \ cdot \ mathbf {\ text {A}} = \ text {BA} \ cos \ theta [/ latex],

, где B — величина магнитного поля (в Тесла, Тл), A — площадь поверхности, а θ — угол между силовыми линиями магнитного поля и нормалью (перпендикулярно) к A.

Для переменного магнитного поля мы сначала рассмотрим магнитный поток [латекс] \ text {d} \ Phi _ \ text {B} [/ latex] через бесконечно малый элемент площади dA, где мы можем считать поле постоянным:

Изменяющееся магнитное поле : Каждая точка на поверхности связана с направлением, называемым нормалью к поверхности; магнитный поток, проходящий через точку, тогда является составляющей магнитного поля вдоль этого нормального направления.

[латекс] \ text {d} \ Phi_ \ text {B} = \ mathbf {\ text {B}} \ cdot \ text {d} \ mathbf {\ text {A}} [/ latex]

Общая поверхность A может быть затем разбита на бесконечно малые элементы, и тогда полный магнитный поток через поверхность равен интегралу поверхности

[латекс] \ Phi_ \ text {B} = \ iint_ \ text {A} \ mathbf {\ text {B}} \ cdot \ text {d} \ mathbf {\ text {A}} [/ latex].

Закон индукции Фарадея и закон Ленца

Закон индукции Фарадея гласит, что ЭДС, вызванная изменением магнитного потока, равна [латексу] \ text {EMF} = — \ text {N} \ frac {\ Delta \ Phi} {\ Delta \ text {t}} [ / латекс], когда поток изменяется на Δ за время Δt.

Цели обучения

Выразите закон индукции Фарадея в форме уравнения

Основные выводы

Ключевые моменты
  • Минус в законе Фарадея означает, что ЭДС создает ток I и магнитное поле B, которые противодействуют изменению потока Δ, известному как закон Ленца.
  • Закон индукции Фарадея является основным принципом работы трансформаторов, индукторов и многих типов электродвигателей, генераторов и соленоидов.
  • Закон Фарадея гласит, что ЭДС, вызванная изменением магнитного потока, зависит от изменения магнитного потока Δ, времени Δt и количества витков катушек.
Ключевые термины
  • электродвижущая сила : (ЭДС) — напряжение, генерируемое батареей или магнитной силой в соответствии с законом Фарадея. Она измеряется в вольтах, а не в ньютонах, и поэтому на самом деле не является силой.
  • Соленоид : Катушка с проволокой, которая действует как магнит, когда через нее протекает электрический ток.
  • поток : Скорость передачи энергии (или другой физической величины) через данную поверхность, в частности электрического или магнитного потока.

Закон индукции Фарадея

Закон индукции Фарадея — это основной закон электромагнетизма, который предсказывает, как магнитное поле будет взаимодействовать с электрической цепью, создавая электродвижущую силу (ЭДС).Это основной принцип работы трансформаторов, индукторов и многих типов электродвигателей, генераторов и соленоидов.

Эксперименты Фарадея показали, что ЭДС, вызванная изменением магнитного потока, зависит только от нескольких факторов. Во-первых, ЭДС прямо пропорциональна изменению потока Δ. Во-вторых, ЭДС является наибольшей, когда изменение во времени Δt наименьшее, то есть ЭДС обратно пропорциональна Δt. Наконец, если катушка имеет N витков, будет создаваться ЭДС, которая в N раз больше, чем для одиночной катушки, так что ЭДС прямо пропорциональна N.Уравнение для ЭДС, вызванной изменением магнитного потока, равно

[латекс] \ text {EMF} = — \ text {N} \ frac {\ Delta \ Phi} {\ Delta \ text {t}} [/ latex].

Это соотношение известно как закон индукции Фарадея. Единицы измерения ЭДС, как обычно, — вольты.

Закон Ленца

Знак минус в законе индукции Фарадея очень важен. Минус означает, что ЭДС создает ток I и магнитное поле B, которые препятствуют изменению потока Δ, известному как закон Ленца. Направление (обозначенное знаком минус) ЭМП настолько важно, что оно названо законом Ленца в честь русского Генриха Ленца (1804–1865), который, подобно Фарадею и Генри, независимо исследовал аспекты индукции.Фарадей знал о направлении, но Ленц указал его, поэтому ему приписывают это открытие.

Закон Ленца : (а) Когда стержневой магнит вставляется в катушку, сила магнитного поля в катушке увеличивается. Ток, наведенный в катушке, создает другое поле в направлении, противоположном стержневому магниту, чтобы противодействовать увеличению. Это один из аспектов закона Ленца: индукция препятствует любому изменению потока. (b) и (c) — две другие ситуации. Убедитесь сами, что показанное направление индуцированной катушки B действительно противостоит изменению магнитного потока и что показанное направление тока согласуется с правилом правой руки.

Энергосбережение

Закон Ленца является проявлением сохранения энергии. Индуцированная ЭДС создает ток, который противодействует изменению потока, потому что изменение потока означает изменение энергии. Энергия может входить или уходить, но не мгновенно. Закон Ленца — это следствие. Когда изменение начинается, закон говорит, что индукция противодействует и, таким образом, замедляет изменение. Фактически, если бы индуцированная ЭДС была в том же направлении, что и изменение потока, была бы положительная обратная связь, которая не давала бы нам бесплатную энергию из любого видимого источника — закон сохранения энергии был бы нарушен.

ЭДС движения

Движение в магнитном поле, которое является стационарным относительно Земли, вызывает ЭДС движения (электродвижущую силу).

Цели обучения

Определить процесс, вызывающий двигательную электродвижущую силу

Основные выводы

Ключевые моменты
  • Закон индукции Фарадея можно использовать для расчета ЭДС движения, когда изменение магнитного потока вызвано движущимся элементом в системе.
  • То, что движущееся магнитное поле создает электрическое поле (и, наоборот, движущееся электрическое поле создает магнитное поле), является частью причины, по которой электрические и магнитные силы теперь рассматриваются как разные проявления одной и той же силы.
  • Любое изменение магнитного потока индуцирует электродвижущую силу (ЭДС), противодействующую этому изменению — процесс, известный как индукция. Движение — одна из основных причин индукции.
Ключевые термины
  • электродвижущая сила : (ЭДС) — напряжение, генерируемое батареей или магнитной силой в соответствии с законом Фарадея. Она измеряется в вольтах, а не в ньютонах, и поэтому на самом деле не является силой.
  • магнитный поток : Мера силы магнитного поля в заданной области.
  • индукция : Генерация электрического тока изменяющимся магнитным полем.

Как было замечено в предыдущих атомах, любое изменение магнитного потока индуцирует электродвижущую силу (ЭДС), противодействующую этому изменению — процесс, известный как индукция. Движение — одна из основных причин индукции. Например, магнит, движущийся к катушке, индуцирует ЭДС, а катушка, движущаяся к магниту, создает аналогичную ЭДС. В этом Атоме мы концентрируемся на движении в магнитном поле, которое является стационарным относительно Земли, производя то, что в общих чертах называется двигательной ЭДС.

Движение ЭДС

Рассмотрим ситуацию, показанную на. Стержень перемещается со скоростью v по паре проводящих рельсов, разделенных расстоянием в однородном магнитном поле B. Рельсы неподвижны относительно B и соединены с неподвижным резистором R ( резистором может быть что угодно от лампочки до вольтметра). Учтите площадь, ограниченную подвижным стержнем, направляющими и резистором. B перпендикулярно этой области, и площадь увеличивается по мере перемещения стержня. Таким образом, магнитный поток между рельсами, стержнем и резистором увеличивается.Когда поток изменяется, ЭДС индуцируется согласно закону индукции Фарадея.

ЭДС движения : (a) ЭДС движения = Bℓv индуцируется между рельсами, когда этот стержень перемещается вправо в однородном магнитном поле. Магнитное поле B направлено внутрь страницы, перпендикулярно движущемуся стержню и рельсам и, следовательно, к области, окружающей их. (б) Закон Ленца дает направление индуцированного поля и тока, а также полярность наведенной ЭДС. Поскольку поток увеличивается, индуцированное поле направлено в противоположном направлении или за пределы страницы.Правило правой руки дает указанное направление тока, и полярность стержня будет управлять таким током.

Чтобы найти величину ЭДС, индуцированной вдоль движущегося стержня, мы используем закон индукции Фарадея без знака:

[латекс] \ text {EMF} = \ text {N} \ frac {\ Delta \ Phi} {\ Delta \ text {t}} [/ latex].

В этом уравнении N = 1 и поток Φ = BAcosθ. У нас θ = 0º и cosθ = 1, так как B перпендикулярно A. Теперь Δ = Δ (BA) = BΔA, поскольку B однородна. Отметим, что площадь, заметаемая стержнем, равна ΔA = ℓx.Ввод этих величин в выражение для ЭДС дает:

[латекс] \ text {EMF} = \ frac {\ text {B} \ Delta \ text {A}} {\ Delta \ text {t}} = \ text {B} \ frac {\ text {l} \ Дельта \ text {x}} {\ Delta \ text {t}} = \ text {Blv} [/ latex].

Чтобы найти направление индуцированного поля, направление тока и полярность наведенной ЭДС, мы применяем закон Ленца, как объяснено в Законе индукции Фарадея: Закон Ленца. Как видно на рис. 1 (b), уровень освещенности увеличивается, так как увеличивается закрытая площадь.Таким образом, индуцированное поле должно противостоять существующему и быть вне страницы. (Правило правой руки требует, чтобы я вращался против часовой стрелки, что, в свою очередь, означает, что вершина стержня положительна, как показано.)

Зависимость электрического поля от магнитного

Между электрической и магнитной силой существует множество связей. То, что движущееся магнитное поле создает электрическое поле (и, наоборот, движущееся электрическое поле создает магнитное поле), является частью причины, по которой электрические и магнитные силы теперь рассматриваются как различных проявлений одной и той же силы (впервые замечено Альбертом Эйнштейном) .Это классическое объединение электрических и магнитных сил в так называемую электромагнитную силу является источником вдохновения для современных усилий по объединению других основных сил.

Обратная ЭДС, вихревые токи и магнитное демпфирование

Обратная ЭДС, вихревые токи и магнитное затухание — все это происходит из-за индуцированной ЭДС и может быть объяснено законом индукции Фарадея.

Цели обучения

Объясните взаимосвязь между двигательной электродвижущей силой, вихревыми токами и магнитным демпфированием

Основные выводы

Ключевые моменты
  • Входной ЭДС, которая питает двигатель, может противодействовать самогенерируемая ЭДС двигателя, называемая обратной ЭДС двигателя.
  • Если ЭДС движения может вызвать токовую петлю в проводнике, ток называется вихревым током.
  • Вихревые токи могут вызывать значительное сопротивление движению, называемое магнитным демпфированием.
Ключевые термины
  • электродвижущая сила : (ЭДС) — напряжение, генерируемое батареей или магнитной силой в соответствии с законом Фарадея. Она измеряется в вольтах, а не в ньютонах, и поэтому на самом деле не является силой.
  • Закон индукции Фарадея : Основной закон электромагнетизма, который предсказывает, как магнитное поле будет взаимодействовать с электрической цепью, создавая электродвижущую силу (ЭДС).

Задний ЭДС

Двигатели и генераторы очень похожи. (Прочтите наши атомы в разделах «Электрические генераторы» и «Электродвигатели».) Генераторы преобразуют механическую энергию в электрическую, а двигатели преобразуют электрическую энергию в механическую. Кроме того, двигатели и генераторы имеют одинаковую конструкцию. Когда катушка двигателя поворачивается, магнитный поток изменяется, и возникает электродвижущая сила (ЭДС), соответствующая закону индукции Фарадея. Таким образом, двигатель действует как генератор всякий раз, когда его катушка вращается.Это произойдет независимо от того, поворачивается ли вал под действием внешнего источника, например ременной передачи, или под действием самого двигателя. То есть, когда двигатель выполняет работу и его вал вращается, возникает ЭДС. Закон Ленца говорит нам, что наведенная ЭДС противодействует любому изменению, так что входной ЭДС, питающей двигатель, будет противодействовать самогенерируемая ЭДС двигателя, называемая обратной ЭДС двигателя.

Вихретоковый

Как обсуждалось в разделе «ЭДС движения», ЭДС движения индуцируется, когда проводник движется в магнитном поле или когда магнитное поле движется относительно проводника.Если подвижная ЭДС может вызвать токовую петлю в проводнике, мы называем этот ток вихревым. Вихревые токи могут вызывать значительное сопротивление движению, называемое магнитным затуханием.

Рассмотрим устройство, показанное на рисунке, которое раскачивает маятник между полюсами сильного магнита. Если боб металлический, то при входе в поле и выходе из поля он испытывает значительное сопротивление, что быстро гасит движение. Однако, если боб представляет собой металлическую пластину с прорезями, как показано на (b), эффект от магнита будет гораздо меньше.Заметного воздействия на боб из изолятора не наблюдается.

Устройство для исследования вихревых токов и магнитного затухания : Обычное демонстрационное устройство для изучения вихревых токов и магнитного затухания. (а) Движение металлического маятника, раскачивающегося между полюсами магнита, быстро затухает под действием вихревых токов. (b) Имеется незначительное влияние на движение металлического боба с прорезями, что означает, что вихревые токи становятся менее эффективными. (c) На непроводящем бобе также отсутствует магнитное затухание, поскольку вихревые токи чрезвычайно малы.

показывает, что происходит с металлической пластиной, когда она входит в магнитное поле и выходит из него. В обоих случаях он испытывает силу, противодействующую его движению. Когда он входит слева, поток увеличивается, и поэтому возникает вихревой ток (закон Фарадея) в направлении против часовой стрелки (закон Ленца), как показано. Только правая сторона токовой петли находится в поле, так что слева на нее действует беспрепятственная сила (правило правой руки). Когда металлическая пластина полностью находится внутри поля, вихревой ток отсутствует, если поле однородно, поскольку поток остается постоянным в этой области.Но когда пластина покидает поле справа, поток уменьшается, вызывая вихревой ток по часовой стрелке, который, опять же, испытывает силу слева, еще больше замедляя движение. Аналогичный анализ того, что происходит, когда пластина поворачивается справа налево, показывает, что ее движение также затухает при входе в поле и выходе из него.

Проводящая пластина, проходящая между полюсами магнита : Более подробный взгляд на проводящую пластину, проходящую между полюсами магнита.Когда он входит в поле и выходит из него, изменение потока создает вихревой ток. Магнитная сила на токовой петле препятствует движению. Когда пластина полностью находится внутри однородного поля, нет ни тока, ни магнитного сопротивления.

Когда металлическая пластина с прорезями входит в поле, как показано на, ЭДС индуцируется изменением магнитного потока, но она менее эффективна, поскольку прорези ограничивают размер токовых петель. Более того, в соседних контурах есть токи в противоположных направлениях, и их эффекты отменяются.Когда используется изолирующий материал, вихревые токи чрезвычайно малы, поэтому магнитное затухание на изоляторах незначительно. Если необходимо избежать вихревых токов в проводниках, они могут быть выполнены с прорезями или состоять из тонких слоев проводящего материала, разделенных изоляционными листами.

Вихревые токи, индуцированные в металлической пластине с прорезями : Вихревые токи, индуцированные в металлической пластине с прорезями, входящие в магнитное поле, образуют небольшие петли, и силы на них имеют тенденцию нейтрализоваться, тем самым делая магнитное сопротивление почти нулевым.

Изменение магнитного потока создает электрическое поле

Закон индукции Фарадея гласит, что изменение магнитного поля создает электрическое поле: [latex] \ varepsilon = — \ frac {\ partial \ Phi_ \ text {B}} {\ partial \ text {t}} [/ latex].

Цели обучения

Опишите взаимосвязь между изменяющимся магнитным полем и электрическим полем

Основные выводы

Ключевые моменты
  • Закон индукции Фарадея — это основной закон электромагнетизма, который предсказывает, как магнитное поле будет взаимодействовать с электрической цепью, создавая электродвижущую силу.
  • Альтернативная дифференциальная форма закона индукции Фарадея выражается в уравнении [латекс] \ nabla \ times \ vec {\ text {E}} = — \ frac {\ partial \ vec {\ text {B}}} { \ partial \ text {t}} [/ latex].
  • Закон индукции Фарадея — одно из четырех уравнений Максвелла, управляющих всеми электромагнитными явлениями.
Ключевые термины
  • векторная область : вектор, величина которого соответствует рассматриваемой области и направление которого перпендикулярно плоскости.
  • Уравнения Максвелла : Набор уравнений, описывающих, как электрические и магнитные поля генерируются и изменяются друг другом, а также зарядами и токами.
  • Теорема Стокса : утверждение об интегрировании дифференциальных форм на многообразиях, которое одновременно упрощает и обобщает несколько теорем векторного исчисления.

Мы изучили закон индукции Фарадея в предыдущих атомах. Мы узнали взаимосвязь между наведенной электродвижущей силой (ЭДС) и магнитным потоком.Вкратце, закон гласит, что изменение магнитного поля [латекс] (\ frac {\ text {d} \ Phi_ \ text {B}} {\ text {dt}}) [/ latex] создает электрическое поле [латекс] (\ varepsilon) [/ latex], закон индукции Фарадея выражается как [latex] \ varepsilon = — \ frac {\ partial \ Phi_ \ text {B}} {\ partial \ text {t}} [/ latex], где [латекс] \ varepsilon [/ latex] — это индуцированная ЭДС, а [latex] \ Phi_ \ text {B} [/ latex] — магнитный поток. («N» опущено из нашего предыдущего выражения. Число витков катушки может быть включено в магнитный поток, поэтому коэффициент не является обязательным.) Закон индукции Фарадея — это основной закон электромагнетизма, который предсказывает, как магнитное поле будет взаимодействовать с электрической цепью, создавая электродвижущую силу (ЭДС). В этом Атоме мы узнаем об альтернативном математическом выражении закона.

Эксперимент Фарадея : эксперимент Фарадея, показывающий индукцию между витками проволоки: жидкая батарея (справа) обеспечивает ток, который течет через небольшую катушку (A), создавая магнитное поле. Когда катушки неподвижны, ток не индуцируется.Но когда малая катушка перемещается внутрь или из большой катушки (B), магнитный поток через большую катушку изменяется, вызывая ток, который регистрируется гальванометром (G).

Дифференциальная форма закона Фарадея

Магнитный поток [латекс] \ Phi_ \ text {B} = \ int_ \ text {S} \ vec {\ text {B}} \ cdot \ text {d} \ vec {\ text {A}} [/ латекс], где [латекс] \ vec {\ text {A}} [/ latex] — это векторная площадь над замкнутой поверхностью S. Устройство, которое может поддерживать разность потенциалов, несмотря на протекание тока, является источником электродвижущей силы. .(EMF) Математически определение [латекс] \ varepsilon = \ oint_ \ text {C} \ vec {\ text {E}} \ cdot \ text {d} \ vec {\ text {s}} [/ latex], где интеграл вычисляется по замкнутому циклу C.

Закон Фарадея теперь можно переписать [latex] \ oint_ \ text {C} \ vec {\ text {E}} \ cdot \ text {d} \ vec {\ text {s}} = — \ frac {\ partial} {\ partial \ text {t}} (\ int \ vec {\ text {B}} \ cdot \ text {d} \ vec {\ text {A}}) [/ latex]. Используя теорему Стокса в векторном исчислении, левая часть равна [latex] \ oint_ \ text {C} \ vec {\ text {E}} \ cdot \ text {d} \ vec {\ text {s}} = \ int_ \ text {S} (\ nabla \ times \ vec {\ text {E}}) \ cdot \ text {d} \ vec {\ text {A}} [/ latex].Также обратите внимание, что в правой части [latex] \ frac {\ partial} {\ partial \ text {t}} (\ int \ vec {\ text {B}} \ cdot \ text {d} \ vec {\ текст {A}}) = \ int \ frac {\ partial \ vec {\ text {B}}} {\ partial \ text {t}} \ cdot \ text {d} \ vec {\ text {A}} [ /латекс]. Таким образом, мы получаем альтернативную форму закона индукции Фарадея: [latex] \ nabla \ times \ vec {\ text {E}} = — \ frac {\ partial \ vec {\ text {B}}} {\ partial \ text {t}} [/ latex]. Это также называют дифференциальной формой закона Фарадея. Это одно из четырех уравнений Максвелла, управляющих всеми электромагнитными явлениями.

Электрогенераторы

Электрические генераторы преобразуют механическую энергию в электрическую; они индуцируют ЭДС, вращая катушку в магнитном поле.

Цели обучения

Объясните, как в электрогенераторах индуцируется электродвижущая сила.

Основные выводы

Ключевые моменты
  • Электрический генератор вращает катушку в магнитном поле, индуцируя ЭДС, заданную как функцию времени величиной ε = NABw sinωt.
  • Генераторы поставляют почти всю мощность для электрических сетей, которые обеспечивают большую часть мировой электроэнергии.
  • Двигатель становится генератором, когда его вал вращается.
Ключевые термины
  • электродвижущая сила : (ЭДС) — напряжение, генерируемое батареей или магнитной силой в соответствии с законом Фарадея. Она измеряется в вольтах, а не в ньютонах, и поэтому на самом деле не является силой.
  • турбина : Любая из различных вращающихся машин, которые используют кинетическую энергию непрерывного потока жидкости (жидкости или газа) для вращения вала.

Электрические генераторы — это устройства, преобразующие механическую энергию в электрическую.Они индуцируют электродвижущую силу (ЭДС), вращая катушку в магнитном поле. Это устройство, преобразующее механическую энергию в электрическую. Генератор заставляет электрический заряд (обычно переносимый электронами) проходить через внешнюю электрическую цепь. Возможные источники механической энергии включают в себя поршневой или турбинный паровой двигатель, воду, падающую через турбину или водяное колесо, двигатель внутреннего сгорания, ветряную турбину, ручной кривошип, сжатый воздух или любой другой источник механической энергии.Генераторы поставляют почти всю мощность для электрических сетей, которые обеспечивают большую часть мировой электроэнергии.

Паровой турбогенератор : современный паротурбинный генератор.

Базовая настройка

Рассмотрим схему, показанную на. Заряды в проводах петли испытывают магнитную силу, потому что они движутся в магнитном поле. Заряды в вертикальных проводах испытывают силы, параллельные проводу, вызывая токи. Однако те, кто находится в верхнем и нижнем сегментах, ощущают силу, перпендикулярную проволоке; эта сила не вызывает тока.Таким образом, мы можем найти наведенную ЭДС, рассматривая только боковые провода. ЭДС движения задается равной ЭДС = Bℓv, где скорость v перпендикулярна магнитному полю B (см. Наш Атом в «ЭДС движения»). Здесь скорость находится под углом θ к B, так что ее составляющая, перпендикулярная B, равна vsinθ.

Схема электрического генератора : Генератор с одной прямоугольной катушкой, вращающейся с постоянной угловой скоростью в однородном магнитном поле, создает ЭДС, синусоидально изменяющуюся во времени.Обратите внимание, что генератор похож на двигатель, за исключением того, что вал вращается для выработки тока, а не наоборот.

Таким образом, в этом случае ЭДС, индуцированная с каждой стороны, равна ЭДС = Bℓvsinθ, и они направлены в одном направлении. Общая ЭДС [латекс] \ varepsilon [/ latex] вокруг петли тогда:

[латекс] \ varepsilon = 2 \ text {Blv} \ sin {\ theta} [/ latex].

Это выражение действительное, но оно не дает ЭДС как функцию времени. Чтобы найти зависимость ЭДС от времени, предположим, что катушка вращается с постоянной угловой скоростью ω.Угол θ связан с угловой скоростью соотношением θ = ωt, так что:

[латекс] \ varepsilon = 2 \ text {Blv} \ sin {\ omega \ text {t}} [/ latex].

Итак, линейная скорость v связана с угловой скоростью соотношением v = rω. Здесь r = w / 2, так что v = (w / 2) ω, и:

[латекс] \ varepsilon = 2 \ text {Bl} \ frac {\ text {w}} {2} \ omega \ sin {\ omega \ text {t}} = (\ text {lw}) \ text {B } \ omega \ sin {\ omega \ text {t}} [/ латекс].

Учитывая, что площадь петли A = ℓw, и учитывая N петель, мы находим, что:

[латекс] \ varepsilon = \ text {NABw} ~ \ sin {\ omega \ text {t}} [/ latex] — это ЭДС, индуцированная в катушке генератора N витков и площади A, вращающейся с постоянной угловой скоростью в однородное магнитное поле B.

Генераторы

, показанные в этом Atom, очень похожи на двигатели, показанные ранее. Это не случайно. Фактически, двигатель становится генератором, когда его вал вращается.

Электродвигатели

Электродвигатель — это устройство, преобразующее электрическую энергию в механическую.

Цели обучения

Объясните, как сила создается в электродвигателях

Основные выводы

Ключевые моменты
  • Большинство электродвигателей используют взаимодействие магнитных полей и токопроводящих проводов для создания силы.
  • Ток в проводнике состоит из движущихся зарядов. Следовательно, катушка с током в магнитном поле также будет ощущать силу Лоренца.
  • В двигателе катушка с током в магнитном поле испытывает силу с обеих сторон катушки, которая создает крутящую силу (называемую крутящим моментом), заставляющую ее вращаться.
Ключевые термины
  • Сила Лоренца : Сила, действующая на заряженную частицу в электромагнитном поле.
  • крутящий момент : вращательное или скручивающее действие силы; (Единица СИ ньютон-метр или Нм; британская единица измерения фут-фунт или фут-фунт)

Основные принципы работы двигателя такие же, как и у генератора, за исключением того, что двигатель преобразует электрическую энергию в механическую энергию (движение).(Сначала прочтите наш атом об электрических генераторах.) Большинство электродвигателей используют взаимодействие магнитных полей и проводников с током для создания силы. Электродвигатели находят применение в самых разных областях, таких как промышленные вентиляторы, нагнетатели и насосы, станки, бытовые приборы, электроинструменты и дисководы.

Лоренц Форс

Если вы поместите движущуюся заряженную частицу в магнитное поле, на нее будет действовать сила, называемая силой Лоренца:

[латекс] \ text {F} = \ text {q} \ times \ text {v} \ times \ text {B} [/ latex]

Правило правой руки : Правило правой руки, показывающее направление силы Лоренца

, где v — скорость движущегося заряда, q — заряд, а B — магнитное поле.Ток в проводнике состоит из движущихся зарядов. Следовательно, катушка с током в магнитном поле также будет ощущать силу Лоренца. Для неподвижного прямолинейного токоведущего провода сила Лоренца составляет:

[латекс] \ text {F} = \ text {I} \ times \ text {L} \ times \ text {B} [/ latex]

, где F — сила (в ньютонах, Н), I — ток в проводе (в амперах, А), L — длина провода, находящегося в магнитном поле (в м). , а B — напряженность магнитного поля (в теслах, Тл).Направление силы Лоренца перпендикулярно как направлению потока тока, так и магнитного поля, и его можно найти с помощью правила правой руки, показанного на рисунке. Используя правую руку, направьте большой палец в направлении тока, и укажите указательным пальцем в направлении магнитного поля. Ваш третий палец теперь будет указывать в направлении силы.

Крутящий момент : Сила на противоположных сторонах катушки будет в противоположных направлениях, потому что заряды движутся в противоположных направлениях.Это означает, что катушка будет вращаться.

Механика двигателя

И двигатели, и генераторы можно объяснить с помощью катушки, вращающейся в магнитном поле. В генераторе катушка подключена к внешней цепи, которая затем включается. Это приводит к изменению потока, который индуцирует электромагнитное поле. В двигателе катушка с током в магнитном поле испытывает силу с обеих сторон катушки, которая создает крутящую силу (называемую крутящим моментом), заставляющую ее вращаться.Любая катушка, по которой проходит ток, может ощущать силу в магнитном поле. Эта сила является силой Лоренца, действующей на движущиеся заряды в проводнике. Сила на противоположных сторонах катушки будет в противоположных направлениях, потому что заряды движутся в противоположных направлениях. Это означает, что катушка будет вращаться.

Индуктивность

Индуктивность — это свойство устройства, которое показывает, насколько эффективно оно индуцирует ЭДС в другом устройстве или на самом себе.

Цели обучения

Описание свойств катушки индуктивности с указанием взаимной индуктивности и самоиндукции

Основные выводы

Ключевые моменты
  • Взаимная индуктивность — это влияние двух устройств, индуцирующих друг в друге ЭДС.Изменение тока ΔI 1 / Δt в одном порождает ЭДС ЭДС2 в секунду: ЭДС 2 = -M ΔI 1 / Δt, где M определяется как взаимная индуктивность между двумя устройствами.
  • Самоиндуктивность — это эффект, который устройство вызывает само по себе.
  • Устройство, которое демонстрирует значительную самоиндукцию, называется индуктором, и ЭДС, индуцированная в нем изменением тока через него, равна ЭДС = −L ΔI / Δt.
Ключевые термины
  • Закон индукции Фарадея : основной закон электромагнетизма, который предсказывает, как магнитное поле будет взаимодействовать с электрической цепью, создавая электродвижущую силу (ЭДС).
  • трансформатор : статическое устройство, которое передает электрическую энергию от одной цепи к другой с помощью магнитной связи. Их основное назначение — передача энергии между различными уровнями напряжения, что позволяет выбирать наиболее подходящее напряжение для выработки, передачи и распределения электроэнергии по отдельности.

Индукция — это процесс, при котором ЭДС индуцируется изменением магнитного потока. Трансформаторы, например, спроектированы так, чтобы быть особенно эффективными для создания желаемого напряжения и тока с очень небольшими потерями энергии в другие формы (см. Наш Atom в разделе «Трансформаторы.«) Есть ли полезная физическая величина, связанная с тем, насколько« эффективно »данное устройство? Ответ — да, и эта физическая величина называется индуктивностью.

Взаимная индуктивность

Взаимная индуктивность — это влияние закона индукции Фарадея для одного устройства на другое, например, первичная катушка, при передаче энергии вторичной обмотке в трансформаторе. Посмотрите, где простые катушки наводят друг на друга ЭДС.

Взаимная индуктивность катушек : Эти катушки могут вызывать ЭДС друг в друге, как неэффективный трансформатор.Их взаимная индуктивность M указывает на эффективность связи между ними. Здесь видно, что изменение тока в катушке 1 вызывает ЭДС в катушке 2. (Обратите внимание, что «E2 индуцированная» представляет наведенную ЭДС в катушке 2.)

Во многих случаях, когда геометрия устройств фиксирована, магнитный поток изменяется за счет изменения тока. Поэтому мы концентрируемся на скорости изменения тока, ΔI / Δt, как на причине индукции. Изменение тока I 1 в одном устройстве, катушка 1, индуцирует ЭДС 2 в другом.Мы выражаем это в форме уравнения как

[латекс] \ text {EMF} _2 = — \ text {M} \ frac {\ Delta \ text {I} _1} {\ Delta \ text {t}} [/ latex],

, где M определяется как взаимная индуктивность между двумя устройствами. Знак минус является выражением закона Ленца. Чем больше взаимная индуктивность M, тем эффективнее связь.

Природа здесь симметрична. Если мы изменим ток I2 в катушке 2, мы индуцируем ЭДС1 в катушке 1, которая равна

[латекс] \ text {EMF} _1 = — \ text {M} \ frac {\ Delta \ text {I} _2} {\ Delta \ text {t}} [/ latex],

, где M то же, что и для обратного процесса.Трансформаторы работают в обратном направлении с такой же эффективностью или взаимной индуктивностью M.

Самоиндуктивность

Самоиндуктивность, действие закона индукции Фарадея устройства на самого себя, также существует. Когда, например, увеличивается ток через катушку, магнитное поле и магнитный поток также увеличиваются, вызывая противоэдс, как того требует закон Ленца. И наоборот, если ток уменьшается, индуцируется ЭДС, препятствующая уменьшению. Большинство устройств имеют фиксированную геометрию, поэтому изменение магнитного потока полностью связано с изменением тока ΔI через устройство.Индуцированная ЭДС связана с физической геометрией устройства и скоростью изменения тока. Выдается

[латекс] \ text {EMF} = — \ text {L} \ frac {\ Delta \ text {I}} {\ Delta \ text {t}} [/ latex],

где L — самоиндукция устройства. Устройство, которое демонстрирует значительную самоиндукцию, называется индуктором. Опять же, знак минус является выражением закона Ленца, указывающего, что ЭДС препятствует изменению тока.

Количественная интерпретация ЭДС движения

A ЭДС движения — это электродвижущая сила (ЭДС), индуцированная движением относительно магнитного поля B.

Цели обучения

Сформулируйте две точки зрения, которые применяются для расчета электродвижущей силы

Основные выводы

Ключевые моменты
  • Движущаяся и наведенная ЭДС — одно и то же явление, только наблюдаемое в разных системах отсчета. Эквивалентность этих двух явлений подтолкнула Эйнштейна к работе над специальной теорией относительности.
  • ЭДС, возникающая из-за относительного движения петли и магнита, определяется как [latex] \ varepsilon _ {\ text {motion}} = \ text {vB} \ times \ text {L} [/ latex] (Eq.1), где L — длина объекта, движущегося со скоростью v относительно магнита.
  • ЭДС можно рассчитать с двух разных точек зрения: 1) с точки зрения магнитной силы, действующей на движущиеся электроны в магнитном поле, и 2) с точки зрения скорости изменения магнитного потока. Оба дают одинаковый результат.
Ключевые термины
  • специальная теория относительности : теория, которая (игнорируя эффекты гравитации) согласовывает принцип относительности с наблюдением, что скорость света постоянна во всех системах отсчета.
  • магнитное поле : Состояние в пространстве вокруг магнита или электрического тока, в котором существует обнаруживаемая магнитная сила и где присутствуют два магнитных полюса.
  • рамка отсчета : система координат или набор осей, в пределах которых можно измерить положение, ориентацию и другие свойства объектов в ней.

Электродвижущая сила (ЭДС), индуцированная движением относительно магнитного поля B, называется ЭДС движения. Вы могли заметить, что ЭДС движения очень похожа на ЭДС, вызванную изменяющимся магнитным полем.В этом атоме мы видим, что это действительно одно и то же явление, показанное в разных системах отсчета.

Движение ЭДС

В случае, когда проводящая петля перемещается в магнит, показанный на (а), магнитная сила, действующая на движущийся заряд в петле, определяется как [латекс] evB [/ латекс] (сила Лоренца, e: заряд электрона).

Петля проводника, движущаяся в магнит : (а) ЭДС движения. Токовая петля переходит в неподвижный магнит. Направление магнитного поля внутрь экрана.(б) Индуцированная ЭДС. Токовая петля неподвижна, а магнит движется.

Из-за силы электроны будут продолжать накапливаться с одной стороны (нижний конец на рисунке), пока на стержне не установится достаточное электрическое поле, препятствующее движению электронов, то есть [латекс] \ text {eE} [/ латекс]. Приравнивая две силы, получаем [латекс] \ text {E} = \ text {vB} [/ latex].

Следовательно, двигательная ЭДС на длине L стороны петли определяется как [latex] \ varepsilon _ {\ text {motion}} = \ text {vB} \ times \ text {L} [/ latex] (Eq .1), где L — длина объекта, движущегося со скоростью v относительно магнита.

Индуцированная ЭДС

Поскольку скорость изменения магнитного потока, проходящего через петлю, равна [latex] \ text {B} \ frac {\ text {dA}} {\ text {dt}} [/ latex] (A: площадь петли что магнитное поле проходит), индуцированная ЭДС [латекс] \ varepsilon _ {\ text {индуцированный}} = \ text {BLv} [/ latex] (уравнение 2).

Эквивалентность движущей и индуцированной ЭДС

Из уравнения. 1 и уравнение. 2 мы можем подтвердить, что двигательная и индуцированная ЭДС дают одинаковый результат.Фактически, эквивалентность двух явлений побудила Альберта Эйнштейна исследовать специальную теорию относительности. В своей основополагающей статье по специальной теории относительности, опубликованной в 1905 году, Эйнштейн начинает с упоминания эквивалентности двух явлений:

«…… например, взаимное электродинамическое действие магнита и проводника. Наблюдаемое явление здесь зависит только от относительного движения проводника и магнита, в то время как обычный взгляд проводит резкое различие между двумя случаями, когда одно или другое из этих тел находится в движении.Ведь если магнит находится в движении, а проводник находится в покое, в окрестности магнита возникает электрическое поле с определенной энергией , производящее ток в местах, где части проводника находятся расположенный. Но если магнит неподвижен, а проводник движется, электрическое поле поблизости от магнита не возникает. В проводнике, однако, мы находим электродвижущую силу, которой сама по себе не соответствует энергия, но которая порождает — при условии равенства относительного движения в двух рассмотренных случаях — электрические токи того же пути и силы, что и создаваемые электрическими силами в первом случае.«

Механические работы и электроэнергия

Механическая работа, совершаемая внешней силой для создания ЭДС движения, преобразуется в тепловую энергию; энергия сохраняется в процессе.

Цели обучения

Применить закон сохранения энергии для описания производственной двигательной электродвижущей силы с механической работой

Основные выводы

Ключевые моменты
  • ЭДС движения, создаваемая движущимся проводником в однородном поле, определяется следующим образом [latex] \ varepsilon = \ text {Blv} [/ latex].
  • Чтобы стержень двигался с постоянной скоростью v, мы должны постоянно прикладывать внешнюю силу F ext к стержню во время его движения.
  • Закон Ленца гарантирует, что движение стержня противоположно, и, следовательно, закон сохранения энергии не нарушается.
Ключевые термины
  • ЭДС движения : ЭДС (электродвижущая сила), индуцированная движением относительно магнитного поля.
  • Закон индукции Фарадея : Основной закон электромагнетизма, который предсказывает, как магнитное поле будет взаимодействовать с электрической цепью, создавая электродвижущую силу (ЭДС).

Мы узнали о двигательной ЭДС ранее (см. Наш Атом в «Двигательной ЭДС»). Для простой схемы, показанной ниже, движущаяся ЭДС [латекс] (\ varepsilon) [/ латекс], создаваемая движущимся проводником (в однородном поле), задается следующим образом:

[латекс] \ varepsilon = \ text {Blv} [/ латекс]

, где B — магнитное поле, l — длина проводящего стержня, а v — (постоянная) скорость его движения. ( B , l и v все перпендикулярны друг другу, как показано на изображении ниже.)

ЭДС движения : (a) ЭДС движения = Bℓv индуцируется между рельсами, когда этот стержень перемещается вправо в однородном магнитном поле. Магнитное поле B направлено внутрь страницы, перпендикулярно движущемуся стержню и рельсам и, следовательно, к области, окружающей их. (б) Закон Ленца дает направление индуцированного поля и тока, а также полярность наведенной ЭДС. Поскольку поток увеличивается, индуцированное поле направлено в противоположном направлении или за пределы страницы. Правило правой руки дает указанное направление тока, и полярность стержня будет управлять таким током.

Сохранение энергии

В этом атоме мы рассмотрим систему с точки зрения энергии . Поскольку стержень движется и пропускает ток и , он ощущает силу Лоренца

.

[латекс] \ text {F} _ \ text {L} = \ text {iBL} [/ latex].

Чтобы стержень двигался с постоянной скоростью v , мы должны постоянно прикладывать внешнюю силу F ext (равную величине F L и противоположную по направлению) к стержню вдоль его движения. .Поскольку стержень движется со скоростью v , мощность P , передаваемая внешней силой, будет:

[латекс] \ text {P} = \ text {F} _ {\ text {ext}} \ text {v} = (\ text {iBL}) \ times \ text {v} = \ text {i} \ варепсилон [/ латекс].

На последнем этапе мы использовали первое уравнение, о котором мы говорили. Обратите внимание, что это в точности мощность, рассеиваемая в контуре (= ток [латекс] \ умноженное на [/ латекс] напряжение). Таким образом, мы заключаем, что механическая работа, совершаемая внешней силой, чтобы стержень двигался с постоянной скоростью, преобразуется в тепловую энергию в контуре.В более общем смысле, механическая работа, совершаемая внешней силой для создания ЭДС движения, преобразуется в тепловую энергию. Энергия сохраняется в процессе.

Закон Ленца

Из «Закона индукции Фарадея и закона Ленца» мы узнали, что закон Ленца является проявлением сохранения энергии. Как мы видим в примере с этим атомом, закон Ленца гарантирует, что движение стержня противодействует из-за склонности природы противодействовать изменению магнитного поля. Если бы индуцированная ЭДС была в том же направлении, что и изменение потока, возникла бы положительная обратная связь, заставляющая стержень улетать от малейшего возмущения.

Энергия в магнитном поле

Магнитное поле накапливает энергию. Плотность энергии задается как [латекс] \ text {u} = \ frac {\ mathbf {\ text {B}} \ cdot \ mathbf {\ text {B}}} {2 \ mu} [/ latex].

Цели обучения

Выразите плотность энергии магнитного поля в форме уравнения

Основные выводы

Ключевые моменты
  • Энергия необходима для создания магнитного поля как для работы против электрического поля, создаваемого изменяющимся магнитным полем, так и для изменения намагниченности любого материала в магнитном поле.2 [/ латекс].
Ключевые термины
  • проницаемость : Количественная мера степени намагничивания материала в присутствии приложенного магнитного поля (измеряется в ньютонах на ампер в квадрате в единицах СИ).
  • индуктор : Пассивное устройство, которое вводит индуктивность в электрическую цепь.
  • ферромагнетик : Материалы, обладающие постоянными магнитными свойствами.

Энергия необходима для создания магнитного поля как для работы против электрического поля, создаваемого изменяющимся магнитным полем, так и для изменения намагниченности любого материала в магнитном поле.Для недисперсионных материалов эта же энергия высвобождается при разрушении магнитного поля. Следовательно, эту энергию можно смоделировать как «хранящуюся» в магнитном поле.

Магнитное поле, создаваемое соленоидом : Магнитное поле, создаваемое соленоидом (вид в разрезе), описанное с использованием силовых линий. Энергия «хранится» в магнитном поле.

Энергия, запасенная в магнитном поле

Для линейных недисперсионных материалов (таких, что B = мкм, H, где мкм, называемая проницаемостью, не зависит от частоты), плотность энергии составляет:

[латекс] \ text {u} = \ frac {\ mathbf {\ text {B}} \ cdot \ mathbf {\ text {B}}} {2 \ mu} = \ frac {\ mu \ mathbf {\ text {H}} \ cdot \ mathbf {\ text {H}}} {2} [/ latex].

Плотность энергии — это количество энергии, хранящейся в данной системе или области пространства на единицу объема. Если поблизости нет магнитных материалов, мкм можно заменить на мкм 0 . Однако приведенное выше уравнение нельзя использовать для нелинейных материалов; необходимо использовать более общее выражение (приведенное ниже).

В общем, дополнительная работа на единицу объема δW , необходимая для того, чтобы вызвать небольшое изменение магнитного поля δ B, составляет:

[латекс] \ delta \ text {W} = \ mathbf {\ text {H}} \ cdot \ delta \ mathbf {\ text {B}} [/ latex].

Когда связь между H и B известна, это уравнение используется для определения работы, необходимой для достижения заданного магнитного состояния. Для гистерезисных материалов, таких как ферромагнетики и сверхпроводники, необходимая работа также зависит от того, как создается магнитное поле. Однако для линейных недисперсионных материалов общее уравнение приводит непосредственно к более простому уравнению плотности энергии, приведенному выше.

Энергия, запасенная в поле соленоида

Энергия, запасенная индуктором, равна количеству работы, необходимой для установления тока через индуктор и, следовательно, магнитного поля.2 [/ латекс].

Трансформаторы

Трансформаторы преобразуют напряжения из одного значения в другое; его функция определяется уравнением трансформатора.

Цели обучения

Примените уравнение трансформатора для сравнения вторичного и первичного напряжений

Основные выводы

Ключевые моменты
  • Трансформаторы часто используются в нескольких точках систем распределения электроэнергии, а также во многих бытовых адаптерах питания.
  • Уравнение трансформатора
  • гласит, что отношение вторичного напряжения к первичному в трансформаторе равно отношению количества витков в их катушках: [латекс] \ frac {\ text {V} _ \ text {s}} {\ text { V} _ \ text {p}} = \ frac {\ text {N} _ \ text {s}} {\ text {N} _ \ text {p}} [/ latex].
  • Если предположить, что сопротивление незначительно, выходная электрическая мощность трансформатора равна его входной. Это приводит нас к другому полезному вопросу: [latex] \ frac {\ text {I} _ \ text {s}} {\ text {I} _ \ text {p}} = \ frac {\ text {N} _ \ текст {p}} {\ text {N} _ \ text {s}} [/ latex]. Если напряжение увеличивается, ток уменьшается. И наоборот, если напряжение уменьшается, ток увеличивается.
Ключевые термины
  • магнитный поток : мера силы магнитного поля в заданной области.
  • Закон индукции Фарадея : Основной закон электромагнетизма, который предсказывает, как магнитное поле будет взаимодействовать с электрической цепью, создавая электродвижущую силу (ЭДС).

Трансформаторы изменяют напряжение с одного значения на другое. Например, такие устройства, как сотовые телефоны, ноутбуки, видеоигры, электроинструменты и небольшая бытовая техника, имеют трансформатор (встроенный в их съемный блок), который преобразует 120 В в напряжение, соответствующее устройству.Трансформаторы также используются в нескольких точках в системах распределения электроэнергии, как показано на рисунке. Мощность передается на большие расстояния при высоком напряжении, поскольку для данного количества мощности требуется меньший ток (это означает меньшие потери в линии). Поскольку высокое напряжение представляет большую опасность, трансформаторы используются для получения более низкого напряжения в месте нахождения пользователя.

Настройка трансформатора : Трансформаторы изменяют напряжение в нескольких точках в системе распределения электроэнергии. Электроэнергия обычно вырабатывается при напряжении более 10 кВ и передается на большие расстояния при напряжениях более 200 кВ, иногда даже 700 кВ, для ограничения потерь энергии.Местное распределение электроэнергии по районам или промышленным предприятиям проходит через подстанцию ​​и передается на короткие расстояния с напряжением от 5 до 13 кВ. Оно снижено до 120, 240 или 480 В для безопасности на месте отдельного пользователя.

Тип трансформатора, рассматриваемого здесь, основан на законе индукции Фарадея и очень похож по конструкции на устройство, которое Фарадей использовал для демонстрации того, что магнитные поля могут создавать токи (показано на рисунке). Две катушки называются первичной и вторичной катушками.При нормальном использовании входное напряжение подается на первичную обмотку, а вторичная обмотка создает преобразованное выходное напряжение. Мало того, что железный сердечник улавливает магнитное поле, создаваемое первичной катушкой, его намагниченность увеличивает напряженность поля. Поскольку входное напряжение переменного тока, изменяющийся во времени магнитный поток направляется во вторичную обмотку, вызывая ее выходное переменное напряжение.

Простой трансформатор : Типичная конструкция простого трансформатора имеет две катушки, намотанные на ферромагнитный сердечник, ламинированный для минимизации вихревых токов.Магнитное поле, создаваемое первичной обмоткой, в основном ограничивается и увеличивается сердечником, который передает его вторичной обмотке. Любое изменение тока в первичной обмотке вызывает ток во вторичной обмотке. На рисунке показан простой трансформатор с двумя катушками, намотанными с обеих сторон многослойного ферромагнитного сердечника. Набор катушек на левой стороне сердечника обозначен как первичный, и его номер указан как N p. Напряжение на первичной обмотке равно V p. Набор катушек на правой стороне сердечника обозначен как вторичный, и его номер представлен как N s.Напряжение на вторичной обмотке равно В с. Символ трансформатора также показан под диаграммой. Он состоит из двух катушек индуктивности, разделенных двумя равными параллельными линиями, представляющими сердечник.

Уравнение трансформатора

Для простого трансформатора, показанного на, выходное напряжение V s почти полностью зависит от входного напряжения V p и соотношения количества петель в первичной и вторичной обмотках. Закон индукции Фарадея для вторичной обмотки дает ее индуцированное выходное напряжение V с как:

[латекс] \ text {V} _ \ text {s} = — \ text {N} _ \ text {s} \ frac {\ Delta \ Phi} {\ Delta \ text {t}} [/ latex],

, где N s — количество витков вторичной катушки, а Δ / Δt — скорость изменения магнитного потока.Обратите внимание, что выходное напряжение равно индуцированной ЭДС (В с = ЭДС с ), при условии, что сопротивление катушки невелико. Площадь поперечного сечения катушек одинакова с обеих сторон, как и напряженность магнитного поля, поэтому / Δt одинаково с обеих сторон. Входное первичное напряжение V p также связано с изменением магнитного потока соотношением:

[латекс] \ text {V} _ \ text {p} = — \ text {N} _ \ text {p} \ frac {\ Delta \ Phi} {\ Delta \ text {t}} [/ latex].

Соотношение этих двух последних уравнений дает полезное соотношение:

[латекс] \ frac {\ text {V} _ \ text {s}} {\ text {V} _ \ text {p}} = \ frac {\ text {N} _ \ text {s}} {\ текст {N} _ \ text {p}} [/ latex].

Это известно как уравнение трансформатора , которое просто утверждает, что отношение вторичного напряжения к первичному в трансформаторе равно отношению количества контуров в их катушках. Выходное напряжение трансформатора может быть меньше, больше или равно входному напряжению, в зависимости от соотношения количества витков в их катушках. Некоторые трансформаторы даже обеспечивают переменный выход, позволяя выполнять подключение в разных точках вторичной обмотки.Повышающий трансформатор — это трансформатор, который увеличивает напряжение, тогда как понижающий трансформатор снижает напряжение.

Если предположить, что сопротивление незначительно, выходная электрическая мощность трансформатора равна его входной. Приравнивание входной и выходной мощности,

[латекс] \ text {P} _ \ text {p} = \ text {I} _ \ text {p} \ text {V} _ \ text {p} = \ text {I} _ \ text {s} \ text {V} _ \ text {s} = \ text {P} _ \ text {s} [/ latex].

Комбинируя эти результаты с уравнением трансформатора, находим:

[латекс] \ frac {\ text {I} _ \ text {s}} {\ text {I} _ \ text {p}} = \ frac {\ text {N} _ \ text {p}} {\ текст {N} _ \ text {s}} [/ latex].

Значит, если напряжение увеличивается, ток уменьшается. И наоборот, если напряжение уменьшается, ток увеличивается.

Формула магнитной индукции

— подробное объяснение и ответы на часто задаваемые вопросы

Магнитная индукция — это явление генерации электродвижущей силы или ЭДС. в проводнике, связанном с изменением связанного с ним магнитного потока. Он был открыт ученым Майклом Фарадеем в 1831 году. Закон индукции Фарадея позже был математически представлен Максвеллом.Магнитная индукция — очень важное научное явление и важнейшая тема в физике. Чтобы понять, что означает формула магнитной индукции, давайте поймем закон индукции Фарадея. Здесь мы также изучим формулу индуцированной ЭДС, формулу закона Фарадея и некоторые другие важные особенности магнитной индукции.

Закон индукции Фарадея

Формула индукции магнитного поля состояний Фарадея путем изменения магнитного потока, связанного с проводником, индуцируется электродвижущая сила (ЭДС).Скорость изменения магнитного потока в замкнутом контуре равна скорости изменения ЭДС.

[Изображение будет загружено в ближайшее время]

Согласно экспериментам Фарадея

ε прямо пропорционально изменению потока

ε обратно пропорционально Δt

ε, полученное в катушке с N витками, в N раз больше, чем одиночная токопроводящая катушка (ε ∝ N)

Магнитный поток, проходящий через поверхность с векторной площадью A:

ΦB = B⋅A = BAcosθ

Для переменного магнитного поля магнитный поток dΦB через бесконечно малую площадь dA :

dΦB = B⋅dA

Поверхностный интеграл дает полный магнитный поток, проходящий через поверхность.

ΦB = ∫∫AB⋅dA

Согласно формуле закона Фарадея, в катушке с N витками, формула, индуцированная ЭДС в замкнутой цепи, определяется как

ЭДС (ε) = — N \ [\ frac {\ Delta \ phi} {\ Delta t} \]

Когда поток изменяется на Δ за время Δt.

Знак минус показывает, что создается ток I и магнитное поле B, противоположное направлению изменения магнитного потока. Это известно как закон Ленца.

Формула электромагнитной индукции для движущегося проводника

[Изображение будет загружено в ближайшее время]

Для движущегося стержня N = 1 и магнитный поток Φ = BAcosθ, θ = 0º и cosθ = 1, a B перпендикулярно A.

Площадь, выметаемая стержнем, равна ΔA = lΔx

∴ ε = \ [\ frac {B \ Delta A} {\ Delta t} \] = \ [\ frac {Bl \ Delta x} {\ Delta t } \] = Blv

, где v (скорость) перпендикулярна B (магнитному полю)

[Изображение будет загружено в ближайшее время]

В приведенном выше сценарии генератора скорость находится под углом θ к B, так что его компонента, перпендикулярная B, равна vsinθ.

ε = Blv sinθ

Где l = длина проводника,

v = скорость проводника

θ = угол между магнитным полем и направлением движения.

Таким образом, формула наведенного тока означает тесную взаимосвязь между электрическим полем и магнитным полем, которая зависит от конкретного изменения во времени.

ЭЛЕКТРОМАГНИТНАЯ ИНДУКЦИЯ

ЭЛЕКТРОМАГНИТНАЯ ИНДУКЦИЯ

На рисунке 32.1 показан стержень из проводящего материала, перемещаемый с помощью скорость v в однородном магнитном поле B. Магнитная сила, действующая на свободную электрон в стержне будет направлен вверх и имеет величину

(32.1)

Рисунок 32.1. Движущийся проводник в магнитном поле. В результате действия магнитной силы электроны начнут накапливаются в верхней части стержня. Распределение заряда стержня будет поэтому измените, и верхушка стержня будет иметь избыток электронов (отрицательный заряд), а нижняя часть стержня будет иметь дефицит электронов (положительный заряд). Это распределение заряда будет создавать электрическое поле в стержень. Напряженность этого электрического поля будет увеличиваться до тех пор, пока электростатическая сила, создаваемая этим полем, равна по величине магнитная сила.В этот момент восходящий поток электронов остановится и

(32,2)

или

(32,3)

Индуцированное электрическое поле будет создавать разность потенциалов [Delta] V между концами стержня, равный

(32,4)

где L — длина стержня. Если концы стержня соединены с цепи, обеспечивающей возврат накопленного заряда, стержень будет источник ЭДС.Поскольку ЭДС связана с движением стержня через магнитное поле называется ЭДС движения . Уравнение (32.4) показывает, что величина ЭДС пропорциональна скорости v. на рисунке 32.1 мы видим, что vL — это площадь, охватываемая стержнем на второй. Величина BvL — это магнитный поток, проходящий через стержень на второй. Таким образом

(32,5)

Хотя эта формула была получена для особого случая, показанного на рисунке 32.1, это действительно в целом. Предназначен для перемещения стержней и проволоки произвольной формы. через произвольные магнитные поля.

Уравнение (32.5) связывает наведенную ЭДС со скоростью, с которой магнитный поток изменяется. В системе, показанной на рисунке 32.1, заключенный флюс изменяется из-за движения стержня. Вложенный магнитный поток также может быть изменяется при изменении напряженности приложенного магнитного поля. В обоих случаях результатом будет наведенная ЭДС.Связь между наведенной ЭДС и изменение магнитного потока известно как закон индукции Фарадея:

«Индуцированная ЭДС при движении или изменении математической траектории при постоянной или изменение магнитного поля равно скорости, с которой магнитный поток проходит через путь. «

Если рассматривать замкнутый путь, закон Фарадея можно сформулировать так:

«Индуцированная ЭДС вокруг замкнутого математического пути в магнитном поле равна к скорости изменения магнитного потока, перехваченного областью в пределах путь «

или

(32.6)

Знак минус в уравнении (32.6) показывает, насколько полярность наведенной ЭДС связаны со знаком потока и скоростью изменения потока. Знак поток фиксируется правилом правой руки:

«Согните пальцы правой руки в том направлении, в котором мы расчет ЭДС вокруг пути; тогда магнитный поток будет положительным, если линии магнитного поля указывают в направлении большого пальца, а отрицательные иначе. «

Пример: проблема 32.13

Металлический стержень длиной L и массой m скользит свободно, без трения, на двух параллельных металлических рельсах. Дорожки соединяются одним концом так, чтобы они и стержень образуют замкнутый контур (см. рисунок 32.2). У стержня есть сопротивление R, и гусеницы имеют незначительное сопротивление. Однородное магнитное поле перпендикулярно плоскости этого контура. Магнитное поле увеличивается при постоянной скорости дБ / дт. Первоначально магнитное поле имеет силу B 0 , а стержень покоится на расстоянии x 0 от соединенный конец рельсов.Выразите ускорение стержня при этом момент в заданных количествах.

Рисунок 32.2. Проблема 32.13.

Магнитный поток [Phi], заключенный между стержнем и дорожками в момент времени t = 0 с, равен выдано

(32,7)

Магнитное поле увеличивается с постоянной скоростью, и, следовательно, закрытый магнитный поток также увеличивается:

(32,8)

Теперь для определения наведенной ЭДС можно использовать закон индукции Фарадея:

(32.9)

В результате наведенной ЭДС через стержень будет протекать ток с величина равна

(32.10)

Направление тока вдоль провода и, следовательно, перпендикулярно к магнитное поле. Сила, действующая на стержень со стороны магнитного поля, равна выдано

(32,11)

(см. главу 31). Комбинируя уравнения (32.10) и (32.11), получаем для силы на провод

(32.12)

Таким образом, ускорение стержня в момент времени t = 0 с равно

. (32,13) ​​

Пример: Задача 32.12

а) Длинный соленоид имеет 300 витков провода на метр и радиус 3,0 см. Если ток в проводе увеличивается со скоростью 50 А / с, то Скорость увеличения напряженности магнитного поля в соленоиде?

б) Соленоид окружен катушкой на 120 витков.Радиус этого катушка 6.0 см. Какая наведенная ЭДС будет генерироваться в этой катушке, пока ток в соленоиде увеличивается?

а) Магнитное поле в соленоиде обсуждалось в главе 31. Если соленоид имеет n витков на метр, и если I — ток через каждую катушку, то поле внутри соленоида равно

(32,14)

Следовательно,

(32,15)

В этой задаче n = 300 витков / метр и dI / dt = 50 A / s.Изменение в магнитное поле, таким образом, равно

(32,16)

Это уравнение показывает, что магнитное поле увеличивается со скоростью 0,019 Т / с.

б) Поскольку магнитное поле в соленоиде меняется, магнитный поток окруженная окружающей катушкой также изменится. Поток, заключенный в одинарная обмотка этой катушки

(32,17)

где r в = 3.0 см — радиус соленоида. Здесь у нас есть Предполагалось, что напряженность магнитного поля вне соленоида равна нулю. Суммарный поток, охватываемый внешними катушками, равен

. (32,18)

Скорость изменения магнитного потока из-за этого изменения магнитного поля равна выдано

(32,19)

В результате изменения тока в соленоиде будет наведена ЭДС во внешней катушке, со значением, равным

(32.20)

Если концы катушки соединены, ток будет течь через дирижер. Направление тока в катушке можно определить с помощью Закон Ленца , который гласит, что

«Индуцированные ЭДС всегда имеют такую ​​полярность, чтобы противодействовать изменение, которое их порождает «

Применим закон Ленца к задаче 32.12. Направление магнитного поля может быть определен с помощью правила правой руки и указывается вправо.Если ток в соленоиде увеличивается, магнитный поток также увеличивается. Электрический ток во внешней катушке будет течь в таком направлении, чтобы противодействовать этому изменению. Это означает, что ток в этой катушке будет течь против часовой стрелки ( поле, создаваемое индуцированным током, направлено противоположно полю генерируется большим соленоидом).

Стержень, движущийся в магнитном поле, будет иметь наведенную ЭДС в результате магнитная сила, действующая на свободные электроны.Индуцированная ЭДС будет пропорциональна линейной скорости v стержня. Если мы посмотрим на стержень из в системе отсчета, в которой стержень покоится, магнитная сила будет равна нулю. Однако все же должна быть наведенная ЭДС. Поскольку эта ЭДС не может быть генерируется магнитным полем, оно должно быть вызвано электрическим полем, которое существует в движущейся системе отсчета. Величина этого электрического поля должен быть таким, чтобы создавалась такая же наведенная ЭДС, что и в система отсчета, в которой движется стержень.Для этого требуется

(32.21)

Электрическое поле E ‘, существующее в системе отсчета движущегося стержня, равно называется индуцированное электрическое поле . ЭДС, возникающая между концы стержня

(32.22)

что эквивалентно уравнению (32.4). Если индуцированное электрическое поле имеет положение зависимой, то мы должны заменить уравнение (32.22) интегральным выражением

(32.23)

где интеграл простирается от одного конца стержня до другого конца стержня. стержень.

Разница между наведенным электрическим полем и электрическим полем генерируется статическим распределением заряда, состоит в том, что в первом случае поле не является консервативным и интеграл по путям по замкнутому пути равен

(32,24)

который не равен нулю, если магнитный поток зависит от времени.

Изменение тока в проводнике (например, в катушке) приводит к изменению магнитного поля. поле.Это зависящее от времени магнитное поле может индуцировать ток за секунду. проводник, если он помещен в это поле. ЭДС, наведенная в эту секунду проводник, [эпсилон] 2 , будет зависеть от магнитного потока через этот проводник:

(32,25)

Поток [Phi] B1 зависит от напряженности магнитного поля. генерируется проводником 1 и, следовательно, пропорционален току I 1 через этот проводник:

(32.26)

Здесь постоянная L 21 зависит от размера двух катушек, от расстояние между ними и количество витков в каждой катушке. В Константа L 21 называется взаимной индуктивностью двух катушек. Используя эту константу, уравнение (32.25) можно переписать как

(32,27)

Единицей индуктивности является Генри (Гн), и из уравнения (32.27) мы заключаем, что

(32,28)

Когда магнитное поле, создаваемое катушкой, изменяется (из-за изменения ток) магнитный поток, заключенный в катушке, также изменится.Это изменение в потоке вызовет ЭДС в катушке, и поскольку ЭДС возникает из-за изменения ток через катушку называется ЭДС самоиндукции. В самоиндуцированная ЭДС равна

(32.29)

В уравнении (32.29) L называется собственной индуктивностью катушки. Самоиндуцированная ЭДС будет действовать в таком направлении, чтобы противодействовать изменению Текущий.

Пример: Задача 32.32

Длинный соленоид радиуса R имеет n витков на единицу длины.Циркуляр катушка из проволоки радиуса R ‘с n’ витками окружает соленоид. Что взаимная индукция? Имеет ли значение форма катушки с проволокой?

Предполагается, что поле внутри соленоида бесконечно длинное. соленоид и имеет силу равную

(32.30)

Поток, заключенный во внешней катушке, равен

(32.31)

Индуцированная ЭДС во внешней катушке равна

(32.32)

Таким образом, взаимная индуктивность L 12 равна

. (32,33)

Если через индуктор протекает постоянный ток, не зависящий от времени магнитный поле создано. Если вдруг источник тока отключится, изменение в заключенном магнитном потоке будет генерировать самоиндуцированную ЭДС, которая будет пытаться чтобы ток продолжал течь в первоначальном направлении. Электроэнергия доставляемая самоиндуцированной ЭДС изначально накапливалась в катушке индуктивности в форма магнитной энергии.Количество магнитной энергии, хранящейся в магнитном поле может быть определено путем расчета общей мощности, передаваемой мощностью источник для создания магнитного поля. Предположим, что после того, как батарея подключенный к катушке индуктивности, ток увеличивается со скоростью dI / dt. В самоиндуцированная ЭДС, создаваемая этим зависящим от времени током, равна

(32,34)

Ток должен обеспечивать дополнительную мощность, чтобы преодолеть эту самоиндуцированную ЭДС. В требуемая мощность будет зависеть от времени и равна

(32.35)

Работа, совершаемая током, сохраняется в индукторе в виде магнитной энергии. В изменение dU в магнитной энергии индуктора, таким образом, равно

(32,36)

Полная энергия, запасенная в магнитном поле индуктора, когда ток достигает своего окончательного значения, может быть получено интегрированием уравнения (32.36) между I = 0 и I = I f .

(32,37)

Для соленоида длиной l собственная индуктивность равна

. (32.38)

Таким образом, магнитная энергия, запасенная в соленоиде, равна

. (32,39)

где V — объем соленоида. Магнитная энергия может быть выражена в условия Б и В:

(32,40)

где B = u 0 n I — магнитное поле в соленоиде. Общая магнитная энергия индуктора теперь может быть выражена через плотность магнитной энергии u, которая определяется как

(32.41)

Магнитная энергия, запасенная в магнитном поле, равна плотности энергии время объем. Хотя мы вывели формулу для магнитной энергии плотности для частного случая очень длинного соленоида, его вывод действителен для любого произвольного магнитного поля.

Пример: Задача 32.46

Тороид квадратного сечения имеет внутренний радиус R 1 и внешний радиус R 2 .Тороид имеет N витков провода, несущего ток I; Предположим, что N очень велико.

а) Найдите плотность магнитной энергии как функцию радиуса.

б) Интегрируя плотность энергии, найдите полную магнитную энергию, хранящуюся в соленоид.

c) Выведите самоиндуктивность по формуле U = L . I 2 /2.

а) Примените закон Ампера, используя сферическую петлю Ампера с радиусом r

(32.42)

Ток, заключенный в амперовскую петлю, равен

. (32,43)

Используя закон Ампера, мы можем определить магнитное поле B:

(32,44)

Таким образом, плотность магнитной энергии равна

(32,45)

б) Пусть высота тороида равна h. Рассмотрим кусочек тороид показан на рисунке 32.3.

Рисунок 32.3. Сечение тороида задачи 32.46. Объем dV этого среза равен

(32,46)

Магнитная энергия, запасенная в этом сегменте, равна

. (32,47)

Полная магнитная энергия, запасенная в тороиде, может быть получена путем интегрирования уравнение (32,47) относительно r между r = 1 и r = 2 :

(32,48)

в) Магнитная энергия, запасенная в индуктивности индуктивности L, равна 0.5 л Я 2 . Сравнивая это с уравнением (32.48), мы заключаем, что собственная индуктивность L тороида равна

(32,49)

Цепь RL состоит из резистора и катушки индуктивности, включенных последовательно с аккумулятор (см. рисунок 32.4). Применяя к этому второму правилу Кричгофа по одноконтурной схеме получаем следующее дифференциальное уравнение

(32.50)

Рисунок 32.4. Схема RL. Это дифференциальное уравнение имеет решение

(32,51)

Это решение действительно, если батарея подключена при t = 0. Уравнение (32.51) показывает, что ток при t = 0 с равен 0 и неуклонно растет, достигая окончательное значение e / R при t = [бесконечность]. Постоянная времени цепи RL равна L / R. Если ток достиг постоянного значения и батарея внезапно разряжена. отключен, проводник может генерировать ток через резистор, который будет постепенно распадаться со временем.Если начальный ток равен [epsilon] / R, ток в момент времени t будет равен

(32,52)

Пример: Задача 32.54

Сколько джоулева тепла рассеивается током в уравнении (32,52) в резистор в интервале времени между t = 0 и t = [бесконечность]? Сравнить с начальная магнитная энергия в индукторе.

Ток через резистор указан в уравнении (32.51). Рассеиваемая мощность по этому ток в резисторе равен

(32.53)

Полная энергия, рассеиваемая этим током в резисторе между t = 0 и t = [бесконечность] равно

(32,54)

Магнитная энергия, запасенная в индукторе, равна

. (32,55)

и мы заключаем, что вся магнитная энергия, запасенная в индукторе, рассеивается как Джоулева тепла в резисторе.


Комментарии, вопросы и / или предложения отправляйте по электронной почте на адрес wolfs @ nsrl.rochester.edu и / или посетите домашнюю страницу Фрэнка Вольфса.

Магнитная индукция

Магнитная индукция
следующий: Motional Emf Up: Магнитная индукция Предыдущая: Закон Ленца


Магнитная индукция Рассмотрим однооборотную петлю из проводящего провода, помещенную в магнитную поле . Петлю связи магнитного потока можно записать
(197)

где любая поверхность, прикрепленная к петле.

Предположим, что магнитное поле изменяется во времени, вызывая цепь связи магнитного потока может меняться. Пусть поток изменится на величину во временном интервале. Согласно закону Фарадея ЭДС индуцированный цикл определяется выражением

(198)

Если есть положительный , тогда ЭДС действует вокруг контура в том же смысле , что и . указал пальцами правой руки, когда большой палец указывает в направлении среднего магнитного поля, проходящего через петлю.Так же, Если есть отрицательный , то ЭДС действует вокруг петли в напротив смысла тому указал пальцами правой руки, когда большой палец указывает в направлении среднего магнитного поля, проходящего через петлю. в В первом случае мы говорим, что ЭДС действует в положительном направлении , тогда как в последнем случае мы говорим, что он действует в отрицательном направлении .

Предположим, что, так что ЭДС действует в положительном направлении.Как именно создается эта ЭДС? Чтобы ответить на этот вопрос, нам нужно напоминать себе, что такое ЭДС на самом деле. Когда мы говорим это ЭДС действует вокруг петли в положительном направлении, на самом деле мы имеем в виду, что заряд, который циркулирует один раз вокруг петля в положительном направлении приобретает энергию. Как заряд получает эту энергию? Понятно, что либо электрическая поле или магнитное поле, или некоторая их комбинация, должна выполнять работать с зарядом, когда он циркулирует по петле.Однако, как мы уже видели, из разд. 8.4, что магнитный поле не может работать с заряженной частицей. Таким образом, заряд должен получать энергию от электрического поля , поскольку оно один раз проходит по петле в положительном направлении.

Согласно разд. 5, работа, которую электрическое поле совершает с зарядом при его движении. петля

(199)

где — линейный элемент цикла.Следовательно, в силу сохранения энергии мы можем написать , или
(200)

Срок в правой части приведенного выше выражения можно распознать как линейный интеграл электрического поля вокруг контура в положительное направление. Таким образом, ЭДС, генерируемая вокруг цепь в положительном направлении равна линейному интегралу электрического поля вокруг цепи в том же направлении.

Уравнения (198) и (200) можно объединить, чтобы получить

(201)

Таким образом, из закона Фарадея следует, что линейный интеграл электрического поля вокруг контура (в положительном направлении) равна минус скорости изменения магнитный поток, связывающий эту цепь.Применяется ли этот закон только к проведению схем, или мы можем применить это к произвольному замкнутому контуру в пространстве? Хорошо, разница между проводящей цепью и произвольной замкнутой петлей в том, что электрический ток может течь по цепи, тогда как ток в общем случае не может обтекать произвольный цикл. Фактически, ЭДС индуцированный вокруг проводящей цепи возбуждает ток вокруг этой цепи, где — сопротивление цепи. Однако, мы можем сделать это сопротивление сколь угодно большим, не нарушая формулу.(201). В пределе, в котором стремится к бесконечности, ток по цепи не течет, таким образом схема становится неотличимой от произвольного цикла. Поскольку мы можем разместить такая схема где-нибудь в космосе, и уравнение. (201) остается в силе, мы вынуждены вывод, что уравнение. (201) действительно для любого замкнутого контура в пространстве, а не только для проводящих цепей.

Уравнение (201) описывает, как изменяющееся во времени магнитное поле генерирует электрическое поле, заполняющее пространство.Напряженность электрического поля прямо пропорциональна скорости изменения магнитного поля. В силовые линии, связанные с этим электрическим поля образуют петли в плоскости, перпендикулярной магнитному полю. Если магнитное поле увеличивается, тогда силовые линии электрического поля циркулируют в противоположное чувство пальцам правой руки, когда большой палец указывает в направлении поля. Если магнитное поле уменьшается, тогда силовые линии электрического поля циркулируют в такое же чувство, как пальцы правой руки, когда большой палец указывает по направлению поля.Это показано на рис. 35.

Рисунок 35: Индуктивно генерируемые электрические поля

Теперь мы можем понять, что когда проводящая цепь помещается в изменяющееся во времени магнитное поле, это электрическое поле, индуцированное изменяющимся магнитным полем. магнитное поле, которое вызывает появление ЭДС вокруг контура. Если в петле есть конечное сопротивление, тогда это электрическое поле также возбуждает ток в цепи.Однако обратите внимание, что электрическое поле создается независимо от наличие токопроводящей цепи. Электрическое поле, создаваемое изменяющимся во времени магнитное поле совершенно отличается по своей природе от поля, создаваемого множеством стационарных электрические заряды. В последнем случае силовые линии электрического поля начинаются на положительные заряды, заканчиваются отрицательными зарядами, и никогда не образуют замкнутых контуров в свободном пространстве. В первом случае силовые линии электрического поля никогда не начинаются или не начинаются. конец, а всегда образуют замкнутые контуры в свободном пространстве.Фактически, электрический силовые линии, создаваемые магнитной индукцией, ведут себя примерно так же как силовые линии магнитного поля. Напомним, из разд. 5.1, что электрический поле, созданное фиксированными расходами, не может выполнять чистую работу по начислению который циркулирует по замкнутому контуру. С другой стороны, электрический поле, создаваемое магнитной индукцией, безусловно, может работать с зарядом который циркулирует по замкнутому контуру. Это в основном то, как индуцируется ток в проводящая петля, помещенная в изменяющееся во времени магнитное поле.Одно из следствий этого факт заключается в том, что работа, выполняемая при медленном перемещении заряда между двумя точками в индуктивном электрическом поле зависит от путь между двумя точками. Отсюда следует, что мы не можем рассчитать уникальную разность потенциалов между двумя точками индуктивного электрическое поле. Фактически, вся идея электрического потенциала разрушается. в таком поле (к счастью, есть способ спасая идею электрического потенциала в индуктивном поле, но эта тема выходит за рамки этого курса).Обратите внимание, однако, что это все еще можно рассчитать уникальное значение для ЭДС, генерируемой вокруг проводящей цепь индуктивным электрическим полем, потому что в этом случае путь, пройденный электрических зарядов однозначно указано: т.е. , заряды имеют следовать схема.



следующий: Motional Emf Up: Магнитная индукция Предыдущая: Закон Ленца
Ричард Фицпатрик 2007-07-14

20.3 Электромагнитная индукция — физика

Изменение магнитного поля

В предыдущем разделе мы узнали, что ток создает магнитное поле. Если природа симметрична, то, возможно, магнитное поле может создать ток. В 1831 году, примерно через 12 лет после открытия, что электрический ток создает магнитное поле, английский ученый Майкл Фарадей (1791–1862) и американский ученый Джозеф Генри (1797–1878) независимо друг от друга продемонстрировали, что магнитные поля могут создавать токи.Основной процесс генерации токов с помощью магнитных полей называется индукцией; этот процесс также называют магнитной индукцией, чтобы отличить его от индукционной зарядки, в которой используется электростатическая кулоновская сила.

Когда Фарадей открыл то, что сейчас называется законом индукции Фарадея, королева Виктория спросила его, как можно использовать электричество. «Мадам, — ответил он, — что хорошего в ребенке?» Сегодня токи, индуцированные магнитными полями, необходимы нашему технологическому обществу. Электрический генератор, который можно найти во всем, от автомобилей до велосипедов и атомных электростанций, использует магнетизм для генерации электрического тока.Другие устройства, которые используют магнетизм для индукции токов, включают в себя звукосниматели в электрогитарах, трансформаторы любого размера, определенные микрофоны, ворота безопасности аэропорта и механизмы демпфирования на чувствительных химических весах.

Один эксперимент, который Фарадей провел для демонстрации магнитной индукции, заключался в том, чтобы переместить стержневой магнит через проволочную катушку и измерить результирующий электрический ток через проволоку. Схема этого эксперимента показана на рис. 20.33. Он обнаружил, что ток индуцируется только тогда, когда магнит движется относительно катушки.Когда магнит неподвижен по отношению к катушке, в катушке не индуцируется ток, как показано на рисунке 20.33. Кроме того, перемещение магнита в противоположном направлении (сравните рис. 20.33 с рис. 20.33) или изменение полярности магнита (сравните рис. 20.33 с рис. 20.33) приводит к возникновению тока в противоположном направлении.

Рисунок 20.33 Движение магнита относительно катушки создает электрические токи, как показано. Такие же токи возникают, если катушку перемещать относительно магнита.Чем больше скорость, тем больше величина тока, и ток равен нулю, когда нет движения. Ток, возникающий при перемещении магнита вверх, имеет направление, противоположное направлению тока, возникающего при перемещении магнита вниз.

Виртуальная физика

Закон Фарадея

Попробуйте это моделирование, чтобы увидеть, как движение магнита создает ток в цепи. Лампочка загорается, чтобы показать, когда течет ток, а вольтметр показывает падение напряжения на лампочке.Попробуйте переместить магнит через четырехвитковую катушку и через двухвитковую катушку. Какая катушка производит более высокое напряжение при одинаковой скорости магнита?

Проверка захвата

Если северный полюс находится влево и магнит перемещается справа налево, при входе магнита в катушку создается положительное напряжение. Какое знаковое напряжение получится, если эксперимент повторить с южным полюсом слева?

  1. Знак напряжения изменится, потому что направление тока изменится при перемещении южного полюса магнита влево.
  2. Знак напряжения останется прежним, потому что направление тока не изменится при перемещении южного полюса магнита влево.
  3. Знак напряжения изменится, потому что величина электрического тока изменится при перемещении южного полюса магнита влево.
  4. Знак напряжения останется прежним, потому что величина тока не изменится при перемещении южного полюса магнита влево.

Индуцированная электродвижущая сила

Если в катушке индуцируется ток, Фарадей рассуждал, что должно быть то, что он назвал электродвижущей силой , проталкивающей заряды через катушку. Эта интерпретация оказалась неверной; вместо этого внешний источник, выполняющий работу по перемещению магнита, добавляет энергию зарядам в катушке. Энергия, добавляемая на единицу заряда, измеряется в вольтах, поэтому электродвижущая сила на самом деле является потенциалом. К сожалению, название «электродвижущая сила» прижилось, а вместе с ним и возможность спутать его с реальной силой.По этой причине мы избегаем термина электродвижущая сила и просто используем сокращение эдс , которое имеет математический символ ε.ε. ЭДС может быть определена как скорость, с которой энергия отбирается от источника на единицу тока, протекающего по цепи. Таким образом, ЭДС — это энергия на единицу заряда , добавляемая источником, что контрастирует с напряжением, которое представляет собой энергию на единицу заряда , высвобождаемую , когда заряды проходят через цепь.

Чтобы понять, почему в катушке возникает ЭДС из-за движущегося магнита, рассмотрим рисунок 20.34, на котором показан стержневой магнит, движущийся вниз относительно проволочной петли. Первоначально через петлю проходят семь силовых линий магнитного поля (см. Изображение слева). Поскольку магнит удаляется от катушки, только пять силовых линий магнитного поля проходят через петлю за короткое время ΔtΔt (см. Изображение справа). Таким образом, когда происходит изменение количества силовых линий магнитного поля, проходящих через область, определяемую проволочной петлей, в проволочной петле индуцируется ЭДС. Подобные эксперименты показывают, что наведенная ЭДС пропорциональна скорости изменения магнитного поля.Математически мы выражаем это как

ε∝ΔBΔt, ε∝ΔBΔt,

20,24

где ΔBΔB — изменение величины магнитного поля за время ΔtΔt, а A — площадь петли.

Рис. 20.34 Стержневой магнит движется вниз относительно проволочной петли, так что количество силовых линий магнитного поля, проходящих через петлю, со временем уменьшается. Это вызывает в контуре ЭДС, создающую электрический ток.

Обратите внимание, что силовые линии магнитного поля, которые лежат в плоскости проволочной петли, на самом деле не проходят через петлю, как показано крайней левой петлей на рисунке 20.35. На этом рисунке стрелка, выходящая из петли, представляет собой вектор, величина которого равна площади петли, а направление перпендикулярно плоскости петли. На рисунке 20.35 петля повернута от θ = 90 ° θ = 90 °. до θ = 0 °, θ = 0 ° вклад силовых линий магнитного поля в ЭДС увеличивается. Таким образом, для создания ЭДС в проволочной петле важна составляющая магнитного поля, которая на перпендикулярна плоскости петли на , то есть Bcosθ.Bcosθ.

Это аналог паруса на ветру.Представьте, что проводящая петля — это парус, а магнитное поле — как ветер. Чтобы максимизировать силу ветра на парусе, парус ориентируют так, чтобы вектор его поверхности указывал в том же направлении, что и ветер, как в самой правой петле на рис. 20.35. Когда парус выровнен так, что вектор его поверхности перпендикулярен ветру, как в крайней левой петле на рис. 20.35, тогда ветер не оказывает силы на парус.

Таким образом, с учетом угла наклона магнитного поля к площади, пропорциональность E∝ΔB / ΔtE∝ΔB / Δt становится равной

E∝ΔBcosθΔt.E∝ΔBcosθΔt.

20,25

Рис. 20.35 Магнитное поле лежит в плоскости крайней левой петли, поэтому в этом случае оно не может генерировать ЭДС. Когда петля поворачивается так, что угол магнитного поля с вектором, перпендикулярным области петли, увеличивается до 90 ° 90 ° (см. Крайнюю правую петлю), магнитное поле вносит максимальный вклад в ЭДС в петле. Точки показывают, где силовые линии магнитного поля пересекают плоскость, определяемую петлей.

Другой способ уменьшить количество силовых линий магнитного поля, проходящих через проводящую петлю на Рисунке 20.35 не для перемещения магнита, а для уменьшения размера петли. Эксперименты показывают, что изменение площади проводящей петли в стабильном магнитном поле вызывает в петле ЭДС. Таким образом, ЭДС, создаваемая в проводящей петле, пропорциональна скорости изменения произведения перпендикулярного магнитного поля и площади петли

. ε∝Δ [(Bcosθ) A] Δt, ε∝Δ [(Bcosθ) A] Δt,

20,26

, где BcosθBcosθ — перпендикулярное магнитное поле, а A — площадь контура.Продукт BAcosθBAcosθ очень важен. Оно пропорционально количеству силовых линий магнитного поля, которые проходят перпендикулярно через поверхность площадью A . Возвращаясь к нашей аналогии с парусом, он будет пропорционален силе ветра на парусе. Он называется магнитным потоком и обозначается как ΦΦ.

Φ = BAcosθΦ = BAcosθ

20,27

Единицей измерения магнитного потока является Вебер (Вб), то есть магнитное поле на единицу площади, или Т / м 2 . Вебер — это также вольт-секунда (Vs).

Индуцированная ЭДС фактически пропорциональна скорости изменения магнитного потока через проводящую петлю.

ε∝ΔΦΔtε∝ΔΦΔt

20,28

Наконец, для катушки, изготовленной из петель Н , ЭДС в Н в раз сильнее, чем для одиночной петли. Таким образом, ЭДС, индуцированная изменяющимся магнитным полем в катушке из N витков , составляет

ε∝NΔBcosθΔtA.ε∝NΔBcosθΔtA.

Последний вопрос, на который нужно ответить, прежде чем мы сможем преобразовать пропорциональность в уравнение: «В каком направлении течет ток?» Русский ученый Генрих Ленц (1804–1865) объяснил, что ток течет в том направлении, которое создает магнитное поле, которое пытается сохранить постоянный поток в контуре.Например, снова рассмотрим рисунок 20.34. Движение стержневого магнита приводит к уменьшению количества направленных вверх силовых линий магнитного поля, которые проходят через петлю. Следовательно, в контуре генерируется ЭДС, которая направляет ток в направлении, которое создает больше направленных вверх линий магнитного поля. Используя правило правой руки, мы видим, что этот ток должен течь в направлении, показанном на рисунке. Чтобы выразить тот факт, что наведенная ЭДС действует как противодействие изменению магнитного потока через проволочную петлю, в пропорциональность ε∝ΔΦ / Δt вводится знак минус.) внутри катушки, направленной влево. Это будет противодействовать увеличению магнитного потока, направленного вправо. Чтобы увидеть, в каком направлении должен течь ток, направьте большой палец правой руки в желаемом направлении магнитного поля B → катушка, B → катушка, и ток будет течь в направлении, указанном сгибанием ваших пальцев правой руки. Это показано изображением правой руки в верхнем ряду рисунка 20.36. Таким образом, ток должен течь в направлении, показанном на рисунке 4 (а).

На Рисунке 4 (b) направление, в котором движется магнит, обратное.В катушке направленное вправо магнитное поле B → magB → mag из-за движущегося магнита уменьшается. Закон Ленца гласит, что, чтобы противостоять этому уменьшению, ЭДС будет управлять током, который создает дополнительное направленное вправо магнитное поле B → катушка B → катушка в катушке. Опять же, направьте большой палец правой руки в желаемом направлении магнитного поля, и ток будет течь в направлении, указанном сгибанием ваших пальцев правой руки (рис. 4 (b)).

Наконец, на Рисунке 4 (c) магнит перевернут, так что южный полюс находится ближе всего к катушке.Теперь магнитное поле B → magB → mag направлено на магнит, а не на катушку. Когда магнит приближается к катушке, он вызывает увеличение направленного влево магнитного поля в катушке. Закон Ленца гласит, что ЭДС, индуцированная в катушке, будет управлять током в направлении, которое создает магнитное поле, направленное вправо. Это будет противодействовать увеличению магнитного потока, направленного влево из-за магнита. Повторное использование правила правой руки, как показано на рисунке, показывает, что ток должен течь в направлении, показанном на рисунке 4 (c).

Рис. 20.36. Закон Ленца гласит, что ЭДС магнитного поля будет управлять током, который сопротивляется изменению магнитного потока в цепи. Это показано на панелях (а) — (с) для различных ориентаций и скоростей магнита. Правые руки справа показывают, как применить правило правой руки, чтобы найти, в каком направлении наведенный ток течет вокруг катушки.

Виртуальная физика

Электромагнитная лаборатория Фарадея

Это моделирование предлагает несколько действий.А пока щелкните вкладку Pickup Coil, которая представляет собой стержневой магнит, который вы можете перемещать через катушку. Когда вы это сделаете, вы увидите, как электроны движутся в катушке, и загорится лампочка, или вольтметр покажет напряжение на резисторе. Обратите внимание, что вольтметр позволяет вам видеть знак напряжения при перемещении магнита. Вы также можете оставить стержневой магнит в покое и переместить катушку, хотя наблюдать за результатами сложнее.

Проверка захвата

Сориентируйте стержневой магнит так, чтобы северный полюс был направлен вправо, и поместите приемную катушку справа от стержневого магнита.Теперь переместите стержневой магнит к катушке и посмотрите, в каком направлении движутся электроны. Это такая же ситуация, как показано ниже. Ток при моделировании течет в том же направлении, что и показано ниже? Объясните, почему да или почему нет.
  1. Да, ток в моделировании течет, как показано, потому что направление тока противоположно направлению потока электронов.
  2. Нет, ток в моделировании течет в противоположном направлении, потому что направление тока совпадает с направлением потока электронов.

Watch Physics

Наведенный ток в проводе

В этом видео объясняется, как можно индуцировать ток в прямом проводе, перемещая его через магнитное поле. Лектор использует перекрестное произведение , которое является типом умножения векторов. Не волнуйтесь, если вы не знакомы с этим, он в основном объединяет правило правой руки для определения силы, действующей на заряды в проводе, с уравнением F = qvBsinθ.F = qvBsinθ.

Проверка захвата

Какая ЭДС создается на прямом проводе 0.через однородное магнитное поле (0,30 Тл) ? Провод проходит в направлении ŷ . Кроме того, какой конец провода имеет более высокий потенциал — пусть нижний конец провода будет на y = 0, а верхний конец на y = 0,5 м)?

  1. 0,15 В и нижний конец провода будет иметь более высокий потенциал
  2. 0,15 В и верхний конец провода будет иметь более высокий потенциал
  3. 0,075 В и нижний конец провода будет иметь более высокий потенциал
  4. 0.075 В и на верхнем конце провода будет более высокий потенциал

Рабочий пример

ЭДС, индуцированная в проводящей катушке движущимся магнитом

Представьте, что магнитное поле проходит через катушку в направлении, указанном на рисунке 20.37. Диаметр катушки 2,0 см. Если магнитное поле изменится с 0,020 до 0,010 Тл за 34 с, каковы направление и величина индуцированного тока? Предположим, что катушка имеет сопротивление 0,1 Ом.

Рисунок 20.37 Катушка, через которую проходит магнитное поле B .

Стратегия

Используйте уравнение ε = −NΔΦ / Δtε = −NΔΦ / Δt, чтобы найти наведенную ЭДС в катушке, где Δt = 34sΔt = 34s. Подсчитав количество петель в соленоиде, находим, что у него 16 петель, поэтому N = 16.N = 16. Используйте уравнение Φ = BAcosθΦ = BAcosθ для расчета магнитного потока

Φ = BAcosθ = Bπ (d2) 2, Φ = BAcosθ = Bπ (d2) 2,

20,30

, где d — диаметр соленоида, а мы использовали cos0 ° = 1. cos0 ° = 1. Поскольку площадь соленоида не меняется, изменение магнитного потока через соленоид составляет

ΔΦ = ΔBπ (d2) 2.ΔΦ = ΔBπ (d2) 2.

20,31

Найдя ЭДС, мы можем использовать закон Ома, ε = IR, ε = IR, чтобы найти ток.

Наконец, закон Ленца гласит, что ток должен создавать магнитное поле, которое препятствует уменьшению приложенного магнитного поля. Таким образом, ток должен создавать магнитное поле справа.

Решение

Объединение уравнений ε = −NΔΦ / Δtε = −NΔΦ / Δt и Φ = BAcosθΦ = BAcosθ дает

ε = −NΔΦΔt = −NΔBπd24Δt.ε = −NΔΦΔt = −NΔBπd24Δt.

20,32

Решение закона Ома для тока и использование этого результата дает

I = εR = −NΔBπd24RΔt = −16 (−0,010T) π (0,020 м) 24 (0,10 Ом) (34 с) = 15 мкА.I = εR = −NΔBπd24RΔt = −16 (−0,010T) π (0,020 м) 24 (0,10 Ом) (34 с) = 15 мкА.

20.33

Закон Ленца гласит, что ток должен создавать магнитное поле справа. Таким образом, мы направляем большой палец правой руки вправо и сжимаем пальцы правой руки вокруг соленоида. Ток должен течь в том направлении, в котором указывают наши пальцы, поэтому он входит в левый конец соленоида и выходит из правого конца.

Обсуждение

Давайте посмотрим, имеет ли смысл знак минус в законе индукции Фарадея. Определите направление магнитного поля как положительное. Это означает, что изменение магнитного поля отрицательное, как мы обнаружили выше. Знак минус в законе индукции Фарадея отрицает отрицательное изменение магнитного поля, оставляя нам положительный ток. Следовательно, ток должен течь в направлении магнитного поля, что мы и обнаружили.

Теперь попробуйте определить положительное направление как направление, противоположное направлению магнитного поля, то есть положительное направление находится слева на рисунке 20.37. В этом случае вы обнаружите отрицательный ток. Но поскольку положительное направление находится влево, отрицательный ток должен течь вправо, что снова согласуется с тем, что мы обнаружили с помощью закона Ленца.

Рабочий пример

Магнитная индукция из-за изменения размера цепи

Схема, показанная на рисунке 20.38, состоит из U-образного провода с резистором, концы которого соединены скользящим токопроводящим стержнем. Магнитное поле, заполняющее область, ограниченную контуром, имеет постоянное значение 0.01 T. Если стержень тянут вправо со скоростью v = 0,50 м / с, v = 0,50 м / с, какой ток индуцируется в цепи и в каком направлении он течет?

Рисунок 20.38 Схема ползунка. Магнитное поле постоянно, и шток тянется вправо со скоростью v . Изменяющаяся область, заключенная в цепи, вызывает в цепи ЭДС.

Стратегия

Мы снова используем закон индукции Фарадея, E = −NΔΦΔt, E = −NΔΦΔt, хотя на этот раз магнитное поле остается постоянным и площадь, ограниченная контуром, изменяется.Схема состоит из одного контура, поэтому N = 1.N = 1. Скорость изменения площади ΔAΔt = vℓ.ΔAΔt = vℓ. Таким образом, скорость изменения магнитного потока равна

ΔΦΔt = Δ (BAcosθ) Δt = BΔAΔt = Bvℓ, ΔΦΔt = Δ (BAcosθ) Δt = BΔAΔt = Bvℓ,

20,34

, где мы использовали тот факт, что угол θθ между вектором площади и магнитным полем равен 0 °. Зная ЭДС, мы можем найти ток, используя закон Ома. Чтобы найти направление тока, мы применяем закон Ленца.

Решение

Закон индукции Фарадея дает

E = −NΔΦΔt = −Bvℓ.E = −NΔΦΔt = −Bvℓ.

20,35

Решение закона Ома для тока и использование предыдущего результата для ЭДС дает

I = ER = −BvℓR = — (0,010T) (0,50 м / с) (0,10 м) 20Ω = 25 мкA I = ER = −BvℓR = — (0,010T) (0,50 м / с) (0,10 м) 20Ω = 25 мкА.

20,36

По мере скольжения стержня вправо магнитный поток, проходящий через контур, увеличивается. Закон Ленца говорит нам, что индуцированный ток создаст магнитное поле, которое будет противодействовать этому увеличению. Таким образом, магнитное поле, создаваемое индуцированным током, должно находиться на странице.Сгибание петли пальцами правой руки по часовой стрелке заставляет большой палец правой руки указывать на страницу, что является желаемым направлением магнитного поля. Таким образом, ток должен течь по цепи по часовой стрелке.

Обсуждение

Сохраняется ли энергия в этой цепи? Внешний агент должен тянуть стержень с достаточной силой, чтобы просто уравновесить силу на проводе с током в магнитном поле — вспомните, что F = IℓBsinθ.F = IℓBsinθ. Скорость, с которой эта сила действует на стержень, должна уравновешиваться скоростью, с которой цепь рассеивает мощность.Используя F = IℓBsinθ, F = IℓBsinθ, сила, необходимая для протягивания проволоки с постоянной скоростью v , равна

. Fpull = IℓBsinθ = IℓB, Fpull = IℓBsinθ = IℓB,

20,37

, где мы использовали тот факт, что угол θθ между током и магнитным полем составляет 90 ° 0,90 °. Подставляя приведенное выше выражение для тока в это уравнение, получаем

Fpull = IℓB = −BvℓR (ℓB) = — B2vℓ2R. Fpull = IℓB = −BvℓR (ℓB) = — B2vℓ2R.

20,38

Сила, создаваемая агентом, тянущим стержень, равна Fpullv, или Fpullv, или

. Потяните = Fpullv = −B2v2ℓ2R.Потяните = Fpullv = −B2v2ℓ2R.

20,39

Мощность, рассеиваемая схемой, составляет

Pdissipated = I2R = (- BvℓR) 2R = B2v2ℓ2R. Pdissipated = I2R = (- BvℓR) 2R = B2v2ℓ2R.

20,40

Таким образом, мы видим, что Ppull + Pdissipated = 0, Ppull + Pdissipated = 0, что означает, что мощность сохраняется в системе, состоящей из цепи и агента, который тянет стержень. Таким образом, в этой системе сохраняется энергия.

Закон электромагнитной индукции Фарадея | Электромагнетизм

10.3 Закон электромагнитной индукции Фарадея (ESBPY)

Ток, индуцированный изменяющимся магнитным полем (ESBPZ)

В то время как удивительное открытие электромагнетизма Эрстедом проложило путь для более практического применения электричества, именно Майкл Фарадей дал нам ключ к практическому производству электричества: электромагнитная индукция .

Фарадей обнаружил, что когда он перемещал магнит рядом с проводом, на нем генерировалось напряжение. Если магнит удерживался в неподвижном состоянии, напряжение не генерировалось, оно существовало только во время движения магнита. Мы называем это напряжение индуцированной ЭДС (\ (\ mathcal {E} \)).

Цепной контур, подключенный к чувствительному амперметру, будет регистрировать ток, если он настроен, как показано на этом рисунке, и магнит перемещается вверх и вниз:

Магнитный поток

Прежде чем мы перейдем к определению закона электромагнитной индукции Фарадея и примерам, нам сначала нужно потратить некоторое время на изучение магнитного потока.Для петли площадью \ (A \) в присутствии однородного магнитного поля \ (\ vec {B} \) магнитный поток (\ (φ \)) определяется как: \ [\ phi = BA \ cos \ theta \] Где: \ begin {align *} \ theta & = \ text {угол между магнитным полем B и нормалью к петле в области A} \\ A & = \ text {область петли} \\ B & = \ text {магнитное поле} \ end {align *}

Единицей измерения магнитного потока является Вебер (Вб).

Вы можете спросить себя, почему включен угол \ (\ theta \). Поток зависит от магнитного поля, проходящего через поверхность. Мы знаем, что поле, параллельное поверхности, не может вызвать ток, потому что оно не проходит через поверхность. Если магнитное поле не перпендикулярно поверхности, то есть компонент, который перпендикулярен, и компонент, который параллелен поверхности. Параллельная составляющая не может вносить вклад в поток, только вертикальная составляющая может.

На этой диаграмме мы показываем, что магнитное поле под углом, отличным от перпендикулярного, может быть разбито на составляющие.Компонент, перпендикулярный поверхности, имеет величину \ (B \ cos (\ theta) \), где \ (\ theta \) — угол между нормалью и магнитным полем.

Закон электромагнитной индукции Фарадея

ЭДС \ (\ mathcal {E} \), создаваемая вокруг контура проводника, пропорциональна скорости изменения магнитного потока φ через площадь A контура. Математически это можно выразить как:

\ [\ mathcal {E} = -N \ frac {\ Delta \ phi} {\ Delta t} \]

где \ (\ phi = B · A \), а B — напряженность магнитного поля.\ (N \) — количество контуров схемы. Магнитное поле измеряется в теслах (Тл). Знак минус указывает направление и то, что наведенная ЭДС имеет тенденцию противодействовать изменению магнитного потока. Знак минус можно не учитывать при вычислении звездных величин.

Закон Фарадея связывает наведенную ЭДС со скоростью изменения магнитного потока, который является произведением магнитного поля и площади поперечного сечения, через которую проходят силовые линии.

Это не площадь самого провода, а площадь, которую он ограничивает.Это означает, что если вы согнете проволоку в круг, площадь, которую мы будем использовать при вычислении потока, будет площадью поверхности круга, а не проволоки.

На этой иллюстрации, где магнит находится в той же плоскости, что и контур цепи, не было бы тока, даже если бы магнит перемещался все ближе и дальше. Это связано с тем, что силовые линии магнитного поля не проходят через замкнутое пространство, а параллельны ему. Силовые линии магнитного поля должны проходить через область, ограниченную контуром цепи, чтобы возникла ЭДС.

Направление индуцированного тока (ESBQ2)

Самая важная вещь, которую следует помнить, это то, что индуцированный ток противодействует происходящему изменению.

На первом рисунке (слева) контурная петля имеет южный полюс приближающегося магнита. Величина поля от магнита становится больше. Реакция наведенной ЭДС будет состоять в том, чтобы попытаться противодействовать усилению поля по направлению к полюсу. Поле является вектором, поэтому ток будет течь в таком направлении, что поля, возникающие из-за тока, имеют тенденцию нейтрализовать поля от магнита, сохраняя результирующее поле неизменным.

Чтобы противостоять переходу от приближающегося южного полюса сверху, ток должен приводить к силовым линиям, удаляющимся от приближающегося полюса. Следовательно, индуцированное магнитное поле должно иметь силовые линии, идущие вниз внутри петли. Направление тока, указанное стрелками на контуре цепи, будет достигнуто. Проверьте это, используя Правило правой руки. Поместите большой палец правой руки в направлении одной из стрелок и обратите внимание на то, что поле закручивается вниз в область, ограниченную петлей.

На второй диаграмме южный полюс удаляется. Это означает, что поле от магнита станет слабее. Отклик на индуцированный ток будет заключаться в создании магнитного поля, которое добавляется к существующему от магнитного поля, чтобы противостоять его уменьшению в силе.

Другой способ представить ту же функцию — просто использовать полюса. Чтобы противостоять приближающемуся южному полюсу, индуцируемый ток создает поле, которое выглядит как другой южный полюс со стороны приближающегося южного полюса.Подобно отталкиванию полюсов, вы можете представить себе, как течение создает южный полюс, чтобы отразить приближающийся южный полюс. На второй панели ток устанавливает северный полюс, чтобы привлечь южный полюс и остановить его движение.

Мы также можем использовать вариант правила правой руки, помещая пальцы в направлении течения, чтобы большой палец указывал в направлении силовых линий (или северного полюса).

Мы можем проверить все это на случаях, когда северный полюс перемещается ближе или дальше от цепи.В первом случае приближения северного полюса ток будет сопротивляться изменению, создавая поле в направлении, противоположном полю, исходящему от магнита, который становится сильнее. Используйте Правило правой руки, чтобы убедиться, что стрелки создают поле с линиями поля, которые изгибаются вверх в замкнутой области, нейтрализуя те, которые изгибаются вниз от северного полюса магнита.

Подобно тому, как полюса отталкиваются, в качестве альтернативы проверьте, что если поместить пальцы правой руки в направлении течения, ваш большой палец будет указывать вверх, указывая на северный полюс.

Для второго рисунка, где северный полюс удаляется, ситуация обратная.

Направление индуцированного тока в соленоиде (ESBQ3)

Подход к просмотру направления тока в соленоиде аналогичен подходу, описанному выше. Единственная разница в том, что в соленоиде есть несколько витков проволоки, поэтому величина наведенной ЭДС будет другой. Поток будет рассчитан с использованием площади поверхности соленоида, умноженной на количество петель.

Помните: направления токов и связанных с ними магнитных полей можно найти, используя только Правило правой руки. Когда пальцы правой руки направлены в направлении магнитного поля, большой палец указывает в направлении тока. Когда большой палец направлен в направлении магнитного поля, пальцы указывают в направлении тока.

Направление тока будет таким, чтобы препятствовать изменению. Мы бы использовали установку, как в этом скетче, для проведения теста:

В случае, когда северный полюс направлен к соленоиду, ток будет течь так, чтобы северный полюс установился на конце соленоида, ближайшем к приближающемуся магниту, чтобы оттолкнуть его (проверьте, используя Правило правой руки):

В случае, когда северный полюс движется от соленоида, ток будет течь так, что южный полюс устанавливается на конце соленоида, ближайшем к удаляющемуся магниту, чтобы притягивать его:

В случае, когда южный полюс движется от соленоида, ток будет течь так, что северный полюс будет установлен на конце соленоида, ближайшем к удаляющемуся магниту, чтобы притягивать его:

В случае, когда южный полюс направлен к соленоиду, ток будет течь так, что южный полюс будет установлен на конце соленоида, ближайшем к приближающемуся магниту, чтобы оттолкнуть его:

Простой способ создать магнитное поле изменяющейся интенсивности — переместить постоянный магнит рядом с проволокой или катушкой с проволокой.Магнитное поле должно увеличиваться или уменьшаться по напряженности перпендикулярно к проводу (так, чтобы силовые линии магнитного поля «пересекали» проводник), иначе не будет индуцироваться напряжение.

Индуцированный ток создает магнитное поле. Индуцированное магнитное поле имеет направление, которое стремится нейтрализовать изменение магнитного поля в петле из проволоки. Итак, вы можете использовать Правило правой руки, чтобы найти направление индуцированного тока, помня, что индуцированное магнитное поле противоположно направлению изменения магнитного поля.

Индукция

Электромагнитная индукция находит практическое применение в конструкции электрических генераторов, которые используют механическую энергию для перемещения магнитного поля мимо катушек с проволокой для генерации напряжения. Однако это далеко не единственное практическое применение этого принципа.

Если мы вспомним, магнитное поле, создаваемое токоведущим проводом, всегда перпендикулярно проводу, и что сила потока этого магнитного поля зависит от величины тока, который проходит через него.Таким образом, мы можем видеть, что провод может создавать напряжение на своей собственной длине , если ток изменяется. Этот эффект называется самоиндукцией . Самоиндукция — это когда изменяющееся магнитное поле создается изменением тока через провод, вызывая напряжение по длине того же провода.

Если магнитный поток усиливается путем сгибания проволоки в форме катушки и / или наматывания этой катушки на материал с высокой проницаемостью, этот эффект самоиндуцированного напряжения будет более интенсивным.Устройство, созданное для использования этого эффекта, называется индуктором .

Помните, что индуцированный ток создает магнитное поле, которое противодействует изменению магнитного потока. Это известно как закон Ленца.

Рабочий пример 1: Закон Фарадея

Рассмотрим плоскую квадратную катушку с 5 витками. Катушка находится в \ (\ text {0,50} \) \ (\ text {m} \) с каждой стороны и имеет магнитное поле \ (\ text {0,5} \) \ (\ text {T} \) проходящий через него. Плоскость катушки перпендикулярна магнитному полю: поле направлено за пределы страницы.Используйте закон Фарадея для вычисления наведенной ЭДС, если магнитное поле увеличивается равномерно от \ (\ text {0,5} \) \ (\ text {T} \) до \ (\ text {1} \) \ (\ текст {T} \) в \ (\ text {10} \) \ (\ text {s} \). Определите направление индуцированного тока.

Определите, что требуется

Мы обязаны использовать Закон Фарадея для расчета наведенной ЭДС.

Запишите закон Фарадея

\ [\ mathcal {E} = — N \ frac {\ Delta \ phi} {\ Delta t} \] Мы знаем, что магнитное поле расположено под прямым углом к ​​поверхности и поэтому выровнено с нормалью.Это означает, что нам не нужно беспокоиться об угле, который поле образует с нормалью и \ (\ phi = BA \). Начальное или начальное магнитное поле, \ (B_i \), задается как конечная величина поля, \ (B_f \). Мы хотим определить величину ЭДС, чтобы можно было игнорировать знак минус.

Площадь \ (A \) — это площадь квадратной катушки. 2 (\ text {1} — \ text {0,50})} {\ text {10}} \\ & = \ текст {0,0625} \ текст {V} \ end {выровнять *}

Наведенный ток направлен против часовой стрелки, если смотреть со стороны нарастающего магнитного поля.

Рабочий пример 2: закон Фарадея

Рассмотрим соленоид из 9 витков с неизвестным радиусом \ (r \). На соленоид действует магнитное поле \ (\ text {0,12} \) \ (\ text {T} \). Ось соленоида параллельна магнитному полю. Когда поле равномерно переключается на \ (\ text {12} \) \ (\ text {T} \) в течение 2 минут, ЭДС величиной \ (- \ text {0,3} \) \ (\ text {V} \) индуцируется. Определите радиус соленоида.

Определите, что требуется

Требуется определить радиус соленоида.Мы знаем, что связь между наведенной ЭДС и полем регулируется законом Фарадея, который включает геометрию соленоида. Мы можем использовать это соотношение, чтобы найти радиус.

Запишите закон Фарадея

\ [\ mathcal {E} = — N \ frac {\ Delta \ phi} {\ Delta t} \] Мы знаем, что магнитное поле расположено под прямым углом к ​​поверхности и поэтому выровнено с нормалью. Это означает, что нам не нужно беспокоиться об угле, который поле образует с нормалью и \ (\ phi = BA \).{- \ text {2}} \) \ (\ text {m} \). Соленоид подвергается воздействию переменного магнитного поля, которое равномерно изменяется от \ (\ text {0,4} \) \ (\ text {T} \) до \ (\ text {3,4} \) \ (\ text { T} \) в интервале \ (\ text {27} \) \ (\ text {s} \). Ось соленоида составляет угол \ (\ text {35} \) \ (\ text {°} \) к магнитному полю. Найдите наведенную ЭДС.

Определите, что требуется

Мы обязаны использовать Закон Фарадея для расчета наведенной ЭДС.

Запишите закон Фарадея

\ [\ mathcal {E} = — N \ frac {\ Delta \ phi} {\ Delta t} \] Мы знаем, что магнитное поле расположено под углом к ​​нормали к поверхности.{- \ text {3}} \ text {V} \ end {выровнять *}

Наведенный ток направлен против часовой стрелки, если смотреть со стороны нарастающего магнитного поля.

Реальные приложения

Следующие устройства используют в своей работе закон Фарадея.

  • индукционные плиты

  • магнитофонов

  • металлоискатели

  • трансформаторы

Реальные применения закона Фарадея

Выберите одно из следующих устройств и поищите в Интернете или библиотеке, как работает ваше устройство.В объяснении вам нужно будет сослаться на закон Фарадея.

  • индукционные плиты

  • магнитофонов

  • металлоискатели

  • трансформаторы

Вы справитесь! Позвольте нам помочь вам учиться с умом для достижения ваших целей. Siyavula Practice поможет вам в удобном для вас темпе, когда вы задаете вопросы в Интернете.

Зарегистрируйтесь, чтобы улучшить свои оценки

Закон Фарадея

Упражнение 10.2

Изложите закон электромагнитной индукции Фарадея словами и запишите математическое соотношение.

ЭДС \ (\ mathcal {E} \), создаваемая вокруг контура проводника, пропорциональна скорости изменения магнитного потока φ через площадь A контура. Математически это можно выразить как:

\ [\ mathcal {E} = -N \ frac {\ Delta \ phi} {\ Delta t} \]

где \ (\ phi = B · A \), а B — напряженность магнитного поля.\ (N \) — количество контуров схемы. Магнитное поле измеряется в теслах (Тл). Знак минус указывает направление и то, что наведенная ЭДС имеет тенденцию противодействовать изменению магнитного потока. Знак минус можно не учитывать при вычислении звездных величин.

Опишите, что происходит, когда стержневой магнит вдавливается в соленоид, подключенный к амперметру, или вытягивается из него. Нарисуйте картинки, подтверждающие ваше описание.

В случае, когда северный полюс направлен к соленоиду, ток будет течь так, чтобы северный полюс установился на конце соленоида, ближайшем к приближающемуся магниту, чтобы оттолкнуть его (проверьте, используя Правило правой руки):

В случае, когда северный полюс движется от соленоида, ток будет течь так, что южный полюс устанавливается на конце соленоида, ближайшем к удаляющемуся магниту, чтобы притягивать его:

В случае, когда южный полюс движется от соленоида, ток будет течь так, что северный полюс будет установлен на конце соленоида, ближайшем к удаляющемуся магниту, чтобы притягивать его:

В случае, когда южный полюс направлен к соленоиду, ток будет течь так, что южный полюс будет установлен на конце соленоида, ближайшем к приближающемуся магниту, чтобы оттолкнуть его:

Объясните, как магнитный поток может быть равен нулю, когда магнитное поле не равно нулю.

Поток связан с магнитным полем:

\ (\ phi = BA \ cos \ theta \)

Если \ (\ cos \ theta \) равно 0, то магнитный поток будет равен 0, даже если есть магнитное поле. В этом случае магнитное поле параллельно поверхности и не проходит через нее.

Используйте правило правой руки, чтобы определить направление индуцированного тока в соленоиде ниже.

Южный полюс магнита приближается к соленоиду.Закон Ленца говорит нам, что ток будет течь, чтобы противодействовать изменению. Южный полюс на конце соленоида будет противодействовать приближающемуся южному полюсу. Ток будет циркулировать по странице в верхней части катушки, так что большой палец правой руки будет указывать влево.

Рассмотрим круговую катушку из 5 витков с радиусом \ (\ text {1,73} \) \ (\ text {m} \). Катушка подвергается воздействию переменного магнитного поля, которое равномерно изменяется от \ (\ text {2,18} \) \ (\ text {T} \) до \ (\ text {12,7} \) \ (\ text { T} \) в интервале \ (\ text {3} \) \ (\ text {minutes} \). {2} & = \ текст {0,0479} \\ г & = \ текст {0,22} \ текст {м} \ end {выровнять *}

Найдите изменение потока, если ЭДС равна \ (\ text {12} \) \ (\ text {V} \) за период \ (\ text {12} \) \ (\ text {s} \) .

\ begin {align *} \ mathcal {E} & = N \ frac {\ Delta \ phi} {\ Delta t} \\ 12 & = 5 \ left (\ frac {\ Delta \ phi} {12} \ right) \\ \ Delta \ phi & = \ text {28,8} \ text {Wb} \ end {выровнять *}

Если угол изменить на \ (\ text {45} \) \ (\ text {°} \), на какой временной интервал нужно изменить, чтобы наведенная ЭДС оставалась прежней?

\ begin {align *} \ mathcal {E} & = N \ frac {\ Delta \ phi} {\ Delta t} \\ & = N \ frac {\ phi_ {f} — \ phi_ {i}} {\ Delta t} \\ & = N \ frac {B_ {f} A \ cos \ theta — B_ {i} A \ cos \ theta} {\ Delta t} \\ & = \ cos \ theta \ times N \ frac {B_ {f} A — B_ {i} A} {\ Delta t} \ end {выровнять *}

Все значения остаются неизменными между двумя описанными ситуациями, за исключением угла и времени.Мы можем приравнять уравнения для двух сценариев:

\ begin {align *} \ mathcal {E} _1 & = \ mathcal {E} _2 \\ \ cos \ theta_1 \ times N \ frac {B_ {f} A — B_ {i} A} {\ Delta t_1} & = \ cos \ theta_2 \ times N \ frac {B_ {f} A — B_ {i} A } {\ Delta t_2} \\ \ cos \ theta_1 \ frac {1} {\ Delta t_1} & = \ cos \ theta_2 \ frac {1} {\ Delta t_2} \\ \ Delta t_2 & = \ frac {\ Delta t_1 \ cos \ theta_2} {\ cos \ theta_1} \\ \ Delta t_2 & = \ frac {(\ text {12} \ cos (\ text {45}} {\ cos (\ text {23})} \\ \ Delta t_2 & = \ text {9,22} \ text {s} \ end {выровнять *}

Электромагнитная индукция

Магнитное поле через петлю можно изменить либо путем изменения величины поля, либо путем изменения площади петли.Чтобы иметь возможность количественно описать эти изменения, магнитный поток определяется как Φ = BA cosθ, где θ — угол между B и направлением, перпендикулярным плоскости петли (вдоль оси петли). .

Закон Фарадея

При изменении магнитного потока через проволочную петлю индуцируется ток. Закон Фарадея гласит, что ЭДС, индуцированная в проводе, пропорциональна скорости потока, проходящего через петлю.Математически

, где N — количество витков, ΔΦ — изменение потока во времени, Δ t . Знак минус указывает полярность наведенной ЭДС.

Предыдущее уравнение легко использовать, когда магнитный поток задается электромагнитом. Если электромагнит включен или выключен, наведенная ЭДС равна количеству витков в контуре, умноженному на скорость изменения магнитного потока. Поток также можно изменить с помощью петли, изменяя размер петли.Представьте скользящую проволоку, как показано на рисунке 1, где l — длина проволоки, которая движется в контакте с U-образной проволокой. В этом случае ε = Blv , где v — скорость длины скольжения.

Обратите внимание, что эта наведенная ЭДС неотличима от ЭДС батареи и что ток по-прежнему является просто скоростью движения зарядов; поэтому закон Ома и другие соотношения для токов в проводах остаются в силе.

Закон Ленца

Направление индуцированного тока можно найти из закона Ленца, который гласит, что магнитное поле, создаваемое индуцированной ЭДС, создает ток, магнитное поле которого противодействует первоначальному изменению потока через проволочную петлю.Снова рассмотрим рисунок и предположим, что слайд движется вправо. Фигуры x указывают на то, что B находится на странице; таким образом, когда слайд перемещается вправо, поле, проходящее через слайд, увеличивается на странице. (Изменение магнитного потока является решающей величиной.) Магнитное поле индуцированного тока будет направлено за пределы страницы, потому что оно будет противодействовать изменению магнитного потока. Воспользуйтесь правилом из вторых рук и поместите изгиб пальцев из страницы в центр петли.Направление большого пальца указывает на то, что ток будет течь против часовой стрелки. (Неправильно утверждать, что ток направлен вправо, потому что он находится слева в верхней части петли.) И наоборот, если ползун перемещается влево, B будет уменьшаться по петле. Изменение потока будет вне страницы, а индуцированный ток будет по часовой стрелке. Тот же анализ используется, если электромагнит включен или выключен.

Рисунок 1

Цепь скользящей проволоки генерирует ЭДС.


Закон Ленца также является законом сохранения. Если бы магнитное поле, создаваемое индуцированным током, могло быть в том же направлении, что и первоначальное изменение магнитного потока, изменение стало бы больше, а индуцированный ток больше. Это невозможное условие было бы лучшим обменом энергии, чем вечный двигатель.

Генераторы и двигатели

Генераторы и двигатели — это применение электромагнитной индукции.На рисунке показан простой электрический генератор.

Рисунок 2

Простой электрогенератор.

Кривошип представляет собой механический метод поворота проволочной петли в магнитном поле. Изменение магнитного потока через петлю генерирует индуцированный ток; таким образом, генератор преобразует механическую энергию в электрическую. Работа двигателя аналогична работе генератора, но в обратном порядке.Двигатель имеет аналогичные физические компоненты, за исключением того, что электрический ток, подаваемый в контур, создает крутящий момент, который поворачивает контур. Таким образом, двигатель преобразует электрическую энергию в механическую.

Взаимная индуктивность и самоиндукция

Взаимная индуктивность возникает, когда две цепи расположены так, что изменение тока в одной вызывает наведение ЭДС в другой.

Представьте себе простую схему переключателя, катушки и батареи.Когда переключатель замкнут, ток через катушку создает магнитное поле. По мере увеличения тока магнитный поток, проходящий через катушку, также изменяется. Этот изменяющийся магнитный поток генерирует ЭДС, противоположную ЭДС батареи. Этот эффект возникает только тогда, когда ток либо увеличивается до своего установившегося значения сразу после замыкания переключателя, либо уменьшается до нуля при размыкании переключателя. Этот эффект называется самоиндукцией . Пропорциональная константа между самоиндуцированной ЭДС и скоростью изменения тока во времени называется индуктивностью (L) и выражается выражением

.

Единицей измерения индуктивности в системе СИ является генри, а 1 генри = 1 (Вс / А).

Используя закон Фарадея, индуктивность можно выразить через изменение магнитного потока и тока:

, где N — количество витков катушки.

Уравнения Максвелла и электромагнитные волны

Уравнения Максвелла суммируют электромагнитные эффекты в четырех уравнениях. Уравнения слишком сложны для этого текста, но концепции, заложенные в них, важно учитывать. Максвелл объяснил, что электрические и магнитные волны могут генерироваться колебаниями электрических зарядов.Эти электромагнитные волны могут быть изображены как скрещенные электрические и магнитные поля, распространяющиеся в пространстве перпендикулярно направлению движения и друг к другу, как показано на рисунке 3.

Рисунок 3

Электромагнитная волна состоит из перпендикулярных колеблющихся магнитного и электрического полей.

Добавить комментарий

Ваш адрес email не будет опубликован. Обязательные поля помечены *

2024 © Все права защищены.